ORTHOPEDIC MCQS WITH ANSWER FOOT 03

ORTHOPEDIC MCQS WITH ANSWER FOOT 03

1.         The main advantage of surgical repair of an acute Achilles tendon rupture, when compared with nonsurgical management, is reduced

 

1-         stiffness of the ankle joint.

2-         risk of deep venous thrombosis.

3-         risk of rerupture.

4-         cost of care.

5-         tendon healing time.

 

PREFERRED RESPONSE: 3

 

DISCUSSION: The literature supports similar clinical outcomes after surgical and nonsurgical methods.  The chief difference lies in the complications between the groups.  Surgical patients experience more wound problems but a significantly lower rerupture rate.  Although suturing the tendon allows earlier mobility, the tendon healing time is unchanged.  Nonsurgical methods are less expensive to provide.

 

REFERENCES: Maffulli N: Rupture of the Achilles tendon.  J Bone Joint Surg Am 1999;81:1019-1036.

Cetti R, Christensen SE, Ejsted R, Jensen NM, Jorgensen U: Operative versus nonoperative treatment of Achilles tendon rupture: A prospective randomized study and review of the literature.  Am J Sports Med 1993;21:791-799.

Nistor L: Surgical and non-surgical treatment of Achilles tendon rupture.  J Bone Joint Surg Am 1981;63:394-399.

 

 

2.        A 25-year-old farm worker sustained a grade III open fracture of the midshaft of the left tibia after falling from a ladder.  Which of the following antibiotic regimens is best for this patient?

 

1-         First-generation cephalosporin

2-         Cephalosporin, penicillin, and quinolone

3-         Cephalosporin and aminoglycoside

4-         Quinolones

5-         Synthetic penicillin and aminoglycoside

 

PREFERRED RESPONSE: 5

 

DISCUSSION: Patients who sustain grade III open fractures that are related to a farm environment require ampicillin or penicillin for Clostridium coverage.

 

REFERENCES: Holton PD, Mader J, Nelson CL, Osmon DR, Patzakis MJ: Antibiotics for the practicing orthopaedic surgeon.  Instr Course Lect 2000;341:36-42.

Wilkins J, Patzakis M: Choice and duration of antibiotics in open fractures.  Orthop Clin North Am 1991;22:433-437.

 

3.        A 60-year-old man reports that he has had shoe pressure pain over his right great toe for several years but has minimal discomfort when barefoot or in sandals.  A clinical photograph and radiographs are shown in Figures 1a through 1c.  Management should consist of

 

1-         cheilectomy.

2-         extra-depth shoes.

3-         steroid injection.

4-         arthrodesis.

5-         joint replacement arthroplasty.

 

PREFERRED RESPONSE: 2

 

DISCUSSION: Some patients have minimal symptoms associated with hallux rigidus despite significant radiographic evidence of osteoarthritis.  This patient’s symptoms are primarily related to shoe pressure from the exostosis and can be managed with extra-depth shoe wear.

 

REFERENCES: Smith RW, Katchis SD, Ayson LC: Outcomes in hallux rigidus patients treated nonoperatively: A long-term follow-up study.  Foot Ankle Int 2000;21:906-913.

Shereff MJ, Baumhauer JF: Hallux rigidus and osteoarthrosis of the first metatarsophalangeal joint.  J Bone Joint Surg Am 1998;80:898-908.

 

4.        A 45-year-old man is seeking evaluation of an injury sustained in a motor vehicle accident 10 weeks ago.  Current radiographs are shown in Figures 2a and 2b.  Based on the radiographic findings, what is the most likely diagnosis?

 

1-         Varus malreduction of the talar neck

2-         Osteonecrosis of the talar body

3-         Subtalar traumatic arthropathy

4-         Nonunion of the talar neck

5-         Occult infection

 

PREFERRED RESPONSE: 2

 

DISCUSSION: An increased density of the talar body compared to the distal tibia following fracture of the talar neck is highly suggestive of vascular compromise of the talar body.  Subchondral osteopenia of the talus at 6 to 8 weeks (Hawkins sign) is a favorable sign but does not eliminate the possibility of osteonecrosis.

 

REFERENCES: Elgafy H, Ebraheim NA, Tile M, Stephen D, Kase J: Fractures of the talus: Experience of two level 1 trauma centers.  Foot Ankle Int 2000;21:1023-1029.

Berlet GC, Lee TH, Massa EG: Talar neck fractures.  Orthop Clin North Am 2001;32:53-64.

 

5.         A 28-year-old woman who is training for the New York Marathon reports pain in the posteromedial aspect of her right ankle.  Examination reveals tenderness just posterior to the medial malleolus.  Radiographs are normal.  An MRI scan is shown in Figure 3.  What is the most likely diagnosis?

 

1-         Posterior tibial tendinitis

2-         Osteoid osteoma

3-         Posterior ankle impingement

4-         Tibial stress fracture

5-         Flexor hallucis longus tendinitis

 

PREFERRED RESPONSE: 4

 

DISCUSSION: Any of the above conditions is credible with a limited history.  The MRI scan unequivocally shows the stress fracture in the distal tibia.  Most tibial stress fractures can be managed with rest and immobilization.

 

REFERENCES: Boden BP, Osbahr DC: High risk stress fractures: Evaluation and treatment.  J Am Acad Orthop Surg 2000;8:344-353.

Lee JK, Yao L: Stress fractures: MR imaging.  Radiology 1988;169:217-220.

 

6.        A 50-year-old laborer sustained an isolated closed injury to his heel after falling 11 feet off a wall.  A radiograph and a CT scan are shown in Figures 4a and 4b.  To minimize the patient’s temporary disability and allow him to return to work most rapidly, management should consist of

 

1-         cast immobilization for 6 weeks, followed by physical therapy.

2-         early range of motion with progressive weight bearing after 6 weeks.

3-         open reduction and internal fixation.

4-         open reduction and primary subtalar arthrodesis.

5-         closed reduction and percutaneous pinning.

 

PREFERRED RESPONSE: 4

 

DISCUSSION: With a severe articular injury to the calcaneus, the ability to achieve satisfactory results with open reduction and internal fixation diminishes.  An arthrodesis is often needed to allow a person who works as a laborer to return to work.  Recent literature suggests that this can be successfully performed primarily, improving the odds of an earlier return to the labor force at 1 year.

 

REFERENCES: Huefner T, Thermann H, Geerling J, Pape HC, Pohlemann T: Primary subtalar arthrodesis of calcaneal fractures.  Foot Ankle Int 2001;22:9-14.

Coughlin MJ: Calcaneal fractures in the industrial patient.  Foot Ankle Int 2000;21:896-905.

Buch BD, Myerson MS, Miller SD: Primary subtalar arthrodesis for the treatment of comminuted calcaneal fractures.  Foot Ankle Int 1996;17:61-70.

 

7.         Figures 5a and 5b show the clinical photograph and radiograph of a patient who has difficulty wearing shoes and has persistent symptoms medially and laterally at the first and fifth metatarsophalangeal joints.  Because shoe modifications have failed to provide relief, management should now consist of

 

1-         bunion repair only.

2-         bunionette repair only with lateral condylectomy.

3-         repair of both with lateral condylectomy.

4-         repair of both with a proximal fifth metatarsal osteotomy.

5-         repair of both with a fifth metatarsal head excision.

 

PREFERRED RESPONSE: 3

 

DISCUSSION: A significant bunionette deformity that fails to respond to conservative management is best addressed surgically, in this case with the bunion deformity.  The radiograph reveals a prominent lateral condyle at the fifth metatarsal head without a significant increase in the intermetatarsal angle.  Simple exostectomy is preferred with less risk of complications.  Complete excision would risk transfer lesions to the medial metatarsals.

 

REFERENCES: Mann RA, Coughlin MJ: Adult hallux valgus, in Coughlin MJ, Mann RA (eds): Surgery of the Foot and Ankle, ed 7.  St Louis, MO, Mosby, 1999, pp 415-435.

Mizel MS, Miller RA, Scioli MW (eds): Orthopaedic Knowledge Update: Foot and Ankle 2.  Rosemont, IL, American Academy of Orthopaedic Surgeons, 1998, pp 163-173.

 

8.         What is the most appropriate orthotic management for the lesion shown in Figure 6?

 

1-         Metatarsal pad

2-         Morton’s extension orthosis

3-         Medial longitudinal arch support

4-         Budin splint

5-         Viscoelastic heel lift

 

PREFERRED RESPONSE: 1

 

DISCUSSION: The figure shows an intractable plantar keratosis (IPK).  The keratoma usually forms beneath a bony prominence.  This can occur under the sesamoids, most commonly the tibial sesamoid, or under the fibular condyle of a prominent metatarsal head.  The initial treatment of an IPK consists of paring down the callused lesion and placing a metatarsal pad proximal to the lesion to provide posting to unload the bony prominence.

 

REFERENCE: Rudicel SA: Intractable plantar keratoses, in Gould J (ed): Operative Foot Surgery.  Philadelphia, PA, WB Saunders, 1994, p 70.

 

9.        Examination of a 45-year-old man with Charcot-Marie-Tooth disease reveals a cavus foot, a tight Achilles tendon, and forefoot callus formation.  Radiographs reveal advanced degenerative changes in the hindfoot.  Shoe wear modifications have failed to provide relief.  Treatment should now consist of

 

1-         triple arthrodesis.

2-         Achilles tendon lengthening and Steindler stripping.

3-         calcaneal osteotomy.

4-         multiple metatarsal osteotomies.

5-         midfoot osteotomy.

 

PREFERRED RESPONSE: 1

 

DISCUSSION: The patient has the typical end stage residuals from long-standing Charcot-Marie-Tooth disease.  Initial management consisting of shoe wear modifications and orthotic devices is preferred, but these are not successful when the disease process has progressed.  Surgical correction with calcaneal osteotomy or Achilles tendon lengthening and Steindler stripping is not indicated in the presence of significant hindfoot arthritis.  Because this patient has findings consistent with hindfoot arthritis, a triple arthrodesis with correction of the cavus deformity is the preferred treatment.

 

REFERENCES: Roper BA, Tibrewal SB: Soft tissue surgery in Charcot-Marie-Tooth disease.  J Bone Joint Surg Br 1989;71:17-20.

Wetmore RS, Drennan JC: Long-term results of triple arthrodesis in Charcot-Marie-Tooth disease.  J Bone Joint Surg Am 1989;71:417-422.

 

10.      A Canale view best visualizes which of the following structures?

 

1-         Posterior facet of the subtalar joint

2-         Lisfranc joint

3-         Talar neck

4-         Sustentaculum tali

5-         Lateral column of the foot

 

PREFERRED RESPONSE: 3

 

DISCUSSION: The Canale view, which visualizes the talar neck, is taken with the ankle in maximum plantar flexion and the foot pronated 15°.  The radiograph is directed at a 75° angle from the horizontal plane in the anteroposterior plane.  The Broden view, which is different from the Canale view, is best for imaging the posterior facet of the subtalar joint.

 

REFERENCES: Canale ST, Kelly FB Jr: Fractures of the neck of the talus: Long-term evaluation of seventy-one cases.  J Bone Joint Surg Am 1978;60:143-156.

Bruden B: Roentgen examination of the subtaloid joint in fractures of the calcaneus.  Acta Radiol 1949;31:85-91.

 

11.       A 45-year-old woman with a long-standing history of diabetes mellitus has a large draining plantar ulcer of the right foot.  Examination reveals some local cellulitis and erythema surrounding the ulcer.  A clinical photograph is shown in Figure 7.  Based on these findings, what is the most appropriate antibiotic?

 

1-         Penicillin G

2-         Cefazolin sodium

3-         Oxacillin

4-         Ampicillin sodium/sulbactam sodium

5-         Gentamicin

 

PREFERRED RESPONSE: 4

 

DISCUSSION: Combination drugs with activity against both aerobic and anaerobic organisms have been determined to be the best approach.  The first-generation cephalosporins do not provide adequate coverage for gram-negative and anaerobic organisms.  Gentamicin alone would not provide adequate activity against anaerobes, and there is the risk of renal and auditory toxicity.

 

REFERENCES: Pinzur MS, Slovenkai MD, Trepman E: Guidelines for diabetic foot care.  Foot Ankle Int 1999;20:695-702.

Eckman MH, Greenfield S, Mackey WC, et al: Foot infections in diabetic patients: Decision and cost-effectiveness analyses.  JAMA 1995;273:712-720.

 

12.      A 35-year-old man reports forefoot pain with weight-bearing activities.  He reports that he has had high arches since adolescence but has never been treated.  Examination reveals stiff cavus feet.  He has no plantar callus or hammer toe formation.  The ankle can be passively dorsiflexed 10°.  Initial management should consist of

 

1-         Achilles tendon lengthening.

2-         calcaneal osteotomy and Steindler stripping.

3-         a molded orthosis.

4-         triple arthrodesis.

5-         metatarsal osteotomy.

 

PREFERRED RESPONSE: 3

 

DISCUSSION: The patient has cavus feet with minimal clinical symptoms.  At this stage, conservative management is preferred.  The use of a molded orthosis will allow better support of the midfoot and provide cushioning of the forefoot.  This will most likely result in long-term relief.  In more advanced cases with forefoot callus formation, Achilles tendon lengthening or calcaneal osteotomy and Steindler stripping are effective in correcting the cavus deformity.  In the presence of arthritic changes in the hindfoot, a triple arthrodesis with corrective bone resection may be necessary.

 

REFERENCES: Janisse DJ: Indications and prescriptions for orthoses in sports.  Orthop Clin North Am 1994;25:95-107.

Franco AH: Pes cavus and pes planus: Analyses and treatment.  Phys Ther 1987;67:688-694.

 

13.      A 70-year-old woman had poliomyelitis as a young child, and the residual weakness she has as an adult principally involves the lower extremities.  She now notes progressive weakness in both legs and she tires easily.  What is the best course of action?

 

1-         Gentle physical therapy and lifestyle modification to reduce demands on the symptomatic muscles

2-         Vigorous physical therapy to rehabilitate the weak muscle groups

3-         Muscle biopsy

4-         MRI of the lumbosacral spine

5-         Electromyography and nerve conduction velocity studies

 

PREFERRED RESPONSE: 1

 

DISCUSSION: The most likely diagnosis is postpolio syndrome, which is characterized by increasing weakness in both the paretic and previously normal muscles.  Fatigability, joint pain, muscle atrophy, respiratory insufficiency, dysphagia, and sleep apnea are also seen.  Gentle exercise and modification in lifestyle demands are generally recommended.  Vigorous rehabilitation is likely to be detrimental in this condition.  Further diagnostic work-up is not indicated at this time.

 

REFERENCES: Dalakas MC, Elder G, Hallett M, et al: A long-term follow-up study of patients with post-poliomyelitis neuromuscular symptoms.  N Eng J Med 1986;314:959-963.

Kasser JE (ed): Orthopaedic Knowledge Update 5.  Rosemont, IL, American Academy of Orthopaedic Surgeons, 1996, pp 683-687.

 

 

 

 

14.      A patient with Charcot-Marie-Tooth disease has a progressively rigid cavovarus foot deformity.  The patient states that the pain is restricted to the forefoot, where rigid claw toe deformities have developed.  Which of the following structures is primarily involved in creation of a claw toe deformity?

 

1-         Laxity of the volar plate

2-         Intraosseous tendon

3-         Extensor digitorum longus tendon

4-         Extensor digitorum brevis tendon

5-         Flexor digitorum longus tendon

 

PREFERRED RESPONSE: 5

 

DISCUSSION: Diseases such as Charcot-Marie-Tooth result in spasticity to the extrinsic flexor tendons.  This results in hyperflexion of the proximal and distal interphalangeal joints of the involved toe, as well as hyperextension at the metatarsophalangeal joint.  The tendon often becomes contracted with progressive equinus of the ankle.  Correction of ankle equinus exaggerates the claw toe deformity.  The interosseous tendon plays no role in the etiology of a claw toe but may become contracted in later stages of the disease.  Laxity of the volar plate may precipitate a claw toe deformity in a nonspastic situation.  In patients with a head injury, claw toe deformities are generally the result of overactivity of the extensor tendons.

 

REFERENCES: Keenan MA, Gorai AP, Smith CW, Garland DE: Intrinsic toe flexion deformity following correction of spastic equinovarus deformity in adults.  Foot Ankle 1987;7:333-337.

Pichney GA, Derner R, Lauf E: Digital “V” arthrodesis.  J Foot Ankle Surg 1993;32:473-479.

Mizel MS, Michelson JD: Nonsurgical treatment of monarticular nontraumatic synovitis of the second metatarsophalangeal joint.  Foot Ankle Int 1997;18:424-426.

 

 

 

 

15.       A 45-year-old man who underwent an ankle arthrodesis reports that for the first 6 years he had significant pain relief after the fusion healed.  However, he now has increasing pain in the sinus tarsi.  AP and lateral radiographs are shown in Figures 8a and 8b.  What is the most likely cause of the patient’s symptoms?

 

1-         Nonunion

2-         Tarsal tunnel syndrome

3-         Degenerative arthritis of the hindfoot joints

4-         Sinus tarsi syndrome

5-         Chronic osteomyelitis of the distal tibia

 

PREFERRED RESPONSE: 3

 

DISCUSSION: The patient has a solid ankle fusion radiographically.  With a tibiotalar arthrodesis, the adjacent joints (subtalar and transverse tarsal) take additional stress.  Over time, progressive degenerative arthritis will occur in these adjacent joints, often necessitating further surgery.

 

REFERENCES: Beaty JH (ed): Orthopaedic Knowledge Update 6.  Rosemont, IL, American Academy of Orthopaedic Surgeons, 1999, pp 613-631.

Coester LM, Saltzman CL, Leupold J, Pontarelli W: Long-term results following ankle arthrodesis for post-traumatic arthritis. J Bone Joint Surg Am 2001;83:219-228.

 

 

 

 

16.      What is the most common surgical cause of the foot deformity shown in Figure 9?

 

1-         Medial tibial sesamoid subluxation

2-         Overcorrection of the intermetatarsal angle

3-         Excessive postoperative dressing application

4-         Excessive medial eminence resection

5-         Excessive lateral soft-tissue release and lateral sesamoidectomy

 

PREFERRED RESPONSE: 5

 

DISCUSSION: The radiograph shows a hallux varus deformity.  Iatrogenically acquired hallux varus is most often the result of excessive lateral soft-tissue release, sesamoidectomy, or both.  It also can be caused by a medial tibial sesamoid subluxation in conjunction with excessive postoperative dressing application, overcorrection of the intermetatarsal angle, or excessive medial eminence resection.

 

REFERENCES: Donley BG: Acquired hallux varus.  Foot Ankle Int 1997;18:586-592.

Myerson MS, Komenda GA: Results of hallux varus correction using an extensor brevis tenodesis.  Foot Ankle Int 1996;17:21-27. 

 

 

 

 

17.       What is the reported failure rate for surgical treatment of a Morton’s neuroma?

 

1-         Less than 1%

2-         5%

3-         15%

4-         30%

5-         50%

 

PREFERRED RESPONSE: 3

 

DISCUSSION: The reported failure rate is in the range of 15%, which may be the result of incorrect diagnosis, improper web space selection, or formation of a stump neuroma.  Therefore, the procedure should be approached with caution, measures should be taken to ensure that the diagnosis is accurate, and nonsurgical options should be exhausted.

 

REFERENCES: Mizel MS, Miller RA, Scioli MW (eds): Orthopaedic Knowledge Update: Foot and Ankle 2.  Rosemont, IL, American Academy of Orthopaedic Surgeons, 1998, pp 101-111.

Beskin JL: Nerve entrapment syndromes of the foot and ankle.  J Am Acad Orthop Surg 1997;5:261-269.

Mann RA, Reynolds JC: Interdigital neuroma: A critical clinical analysis.  Foot Ankle 1983;3:238-243.

 

 

 

 

18.       A 35-year-old woman who runs long distance has had posterior calf tenderness for the past 3 months.  A clinical photograph is shown in Figure 10a, and MRI scans are shown in Figures 10b and 10c.  Management at this point should consist of

 

1-         a non-weight-bearing cast for 4 weeks.

2-         eccentric calf stretching and physical therapy.

3-         a cortisone injection.

4-         tendon debridement.

5-         tendon debridement and augmentation.

 

PREFERRED RESPONSE: 2

 

DISCUSSION: The initial treatment for peritendinitis should consist of calf stretching in an eccentric mode and physical therapy.  In a recent study, this treatment has been found superior to surgical debridement in nonextensive peritendinitis and pantendinitis.  A non-weight-bearing cast, while useful in reducing inflammation, will result in calf atrophy and poorly organized collagen repair.  Cortisone is contraindicated because of the danger of tendon damage.  Tendon debridement at this stage is not indicated.

 

REFERENCES: Alfredson H, Pietila T, Jansson P, Lorentzon R: Heavy-load eccentric calf muscle training for the treatment of chronic Achilles tendinosis.  Am J Sports Med 1998;26:360-366.

Angermann P, Hougaard D: Chronic Achilles tendinopathy in athletic individuals: Results of nonsurgical treatment.  Foot Ankle Int 1999;20:304-306.

 

19.      A 35-year-old laborer who sustained a forefoot injury 10 years ago has returned to work but reports a progressively painful deformity of the hallux and continued midfoot pain that is aggravated by weight-bearing activities.  Shoe wear modifications have failed to provide relief. Direct palpation reveals no pain at the first metatarsocuneiform joint.   A radiograph is shown in Figure 11.  What is the next most appropriate step in management?

 

1-         Open treatment of the metatarsal malunion

2-         Open treatment of the metatarsal nonunions and hallux metatarsophalangeal arthrodesis

3-         Cast application and an external bone stimulator

4-         Hallux valgus correction

5-         Hallux metatarsophalangeal arthrodesis

 

PREFERRED RESPONSE: 2

 

DISCUSSION: The patient has nonunions of the metatarsal fractures and a hallux valgus deformity with arthritic changes.  To address all of the findings, management should consist of open treatment of the metatarsal nonunions and hallux metatarsophalangeal arthrodesis.  Cast immobilization and a bone stimulator are unlikely to be beneficial at this time.  Isolated correction of the hallux valgus deformity will not address the metatarsal nonunions or the arthritis at the hallux metatarsophalangeal joint.

 

REFERENCES: Kitaoka HB, Patzer GL: Arthrodesis versus resection arthroplasty for failed hallux valgus operations.  Clin Orthop 1998;347:208-214. 

McGarvey WC, Braly WG: Bone graft in hindfoot arthrodesis: Allograft vs autograft.  Orthopedics 1996;19:389-394. 

Ouzounian TJ: Metatarsophalangeal arthrodesis for salvage of failed hallux valgus surgery.  Foot Ankle Clin 1997;2:741-752.

 

 

 

 

20.      The use of posting (a wedge added to the medial or lateral side of an insole) is useful to balance forefoot or hindfoot malalignment.  Assuming normal subtalar joint pronation, what is the maximum amount of recommended hindfoot posting?

 

1-         0 degrees

2-         1 degrees to 2 degrees

3-         Approximately 5 degrees

4-         Approximately 10 degrees

5-         10 degrees to 15 degrees

 

PREFERRED RESPONSE: 3

 

DISCUSSION: Generally, patients cannot tolerate more than 5 degrees of hindfoot posting.

 

REFERENCES: Donatelli RA, Hurlbert C, Conaway D, et al: Biomechanics foot orthotics: A retrospective study.  J Orthop Sports Phys Ther 1988;10:205-212.

Michaud TM: Foot Orthoses and Other Forms of Conservative Foot Care. Baltimore, MD, Williams & Wilkins, 1993, pp 61-65, 186.

Root MC, Orien WP, Weed JH: Normal and Abnormal Function of the Foot.  Los Angeles, CA, Clinical Biomechanics Corporation, 1977, vol 2, pp 29-33.

 

 

 

 

21.      A 40-year-old woman has a symptomatic mass on the anterior aspect of the ankle.  She reports no constitutional symptoms.  An MRI scan is shown in Figure 12.  What is the most likely diagnosis?

 

1-         Ganglion

2-         Osteosarcoma

3-         Aneurysmal bone cyst

4-         Unicameral bone cyst

5-         Gouty tophi

 

PREFERRED RESPONSE: 1

 

DISCUSSION: The MRI scan reveals a lobular mass that is below the vitamin E tablet marker taped to the skin.  This is juxtaposed to the tibialis anterior tendon.  It is slightly more enhanced than the surrounding subcutaneous fat and is consistent with a ganglion.  Osteosarcoma, aneurysmal bone cyst, or unicameral bone cyst all would demonstrate enhancement or pathology in the bone.  This is clearly a well-defined soft-tissue mass.  Gouty tophi show low to intermediate signal on T1- and T2-weighted images.

 

REFERENCES: Kransdorf MJ, Jelinek JS, Moser RP Jr, et al: Soft tissue masses: Diagnosis using MR imaging.  Am J Roentgenol 1989;153:541-547.

Wetzel LH, Levine E: Soft-tissue tumors of the foot: Value of MR imaging for specific diagnosis.  Am J Roentgenol 1990;155:1025-1030.

Gerster JC, Landry M, Dufresne L, Meuwly JY: Imaging of tophaceous gout: Computed tomography provide specific images compared with magnetic resonance imaging and ultrasonography.  Ann Rheum Dis 2002;61:52-54.

 

 

 

 

22.      A 35-year-old woman who is training for a triathlon has had a 2-month history of heel pain with weight bearing and is unable to run.  History reveals that she is amenorrheic.  Examination reveals that she is thin and has pain over the heel that is exacerbated with medial and lateral compression.  Range of motion and motor and sensory function are normal.  Radiographs are normal.  What is the most likely diagnosis?

 

1-         Plantar fasciitis

2-         Seronegative inflammatory arthritis

3-         Stress fracture of the calcaneus

4-         Tarsal tunnel syndrome

5-         Peripheral neuropathy

 

PREFERRED RESPONSE: 3

 

DISCUSSION: The most likely diagnosis is a stress fracture of the calcaneus and is supported by the history of running, female gender, and amenorrhea.  Reproducing pain with medial and lateral compression of the heel also supports the diagnosis.  A bone scan or MRI would most likely confirm the diagnosis.  Plantar fasciitis would result in pain on the bottom of the heel with point tenderness.  The lack of other areas of involvement or other symptoms does not support a seronegative inflammatory arthritis.  Tarsal tunnel syndrome and peripheral neuropathy are unlikely because of the normal neurologic examination.

 

REFERENCES: Beaty JH (ed): Orthopaedic Knowledge Update 6.  Rosemont, IL, American Academy of Orthopaedic Surgeons, 1999, pp 597-612.

Lutter LD, Mizel MS, Pfeffer GB (eds): Orthopaedic Knowledge Update: Foot and Ankle.  Rosemont, IL, American Academy of Orthopaedic Surgeons, 1994, pp 179-191.

 

23.      A 28-year-old woman has a moderate hallux valgus deformity and a prominence of the medial eminence.  She can participate in all activities and reports that she could wear 3-inch heels in the past, but she now notes medial eminence pain even while wearing a soft leather flat shoe with a cushioned sole.  She requests recommendations regarding surgical correction.  Examination reveals a 1-2 intermetatarsal angle of 10 degrees.  A clinical photograph and radiograph are shown in Figures 13a and 13b.  What is the best course of action?

 

1-         Chevron osteotomy to correct hallux valgus

2-         Custom orthosis to prevent further deformity

3-         Observation only

4-         Steroid injection to decrease inflammation

5-         Extra-depth shoes

 

PREFERRED RESPONSE: 1

 

DISCUSSION: Based on her symptoms and prior shoe wear modifications, the treatment of choice is surgical correction of the hallux valgus with a chevron osteotomy.  There are no data to support the use of a custom orthosis to delay the progression of a hallux valgus deformity.  Steroid injection would only risk infection, as well as joint and capsule damage.  Extra-depth shoes are an option; however, the patient is interested in surgical options.

 

REFERENCES: Chou LB, Mann RA, Casillas MM: Biplanar chevron osteotomy.  Foot Ankle Int 1998;19:579-584.

Coughlin MJ: Roger A. Mann Award: Juvenile hallux valgus. Etiology and treatment.  Foot Ankle Int 1995;16:682-697.

Pochatko DJ, Schlehr FJ, Murphey MD, Hamilton JJ: Distal chevron osteotomy with lateral release for treatment of hallux valgus deformity.  Foot Ankle Int 1994;15:457-461.

 

 

 

 

24.      A 20-year-old woman has lateral foot and ankle pain after sustaining an inversion injury of the ankle while playing soccer 3 months ago.  Activity modifications and physical therapy have failed to provide relief.  She describes burning pain that extends from the anterior aspect of the ankle to the foot and lateral two toes.  The pain is often worse at night.  Plain radiographs, a bone scan, and an MRI scan are normal.  Stress examination reveals no instability.  What is the most likely diagnosis?

 

1-         Occult traumatic osteochondral lesion of the lateral talar dome

2-         Neurapraxia of the intermediate branch of the superficial peroneal nerve

3-         Incompetence of the superior peroneal retinaculum with peroneal subluxation

4-         Meniscoid lesion of the anterior talofibular ligament

5-         Syndesmotic injury (high ankle sprain)

 

PREFERRED RESPONSE: 2

 

DISCUSSION: Persistent pain following an ankle sprain can present a diagnostic dilemma.  All of the injuries listed should be considered in the differential diagnosis.  The superficial peroneal nerve courses in the lateral compartment and exits the crural fascia 12 to 15 cm above the level of the ankle.  Muscle herniation through the fascial defect has been reported to be associated with entrapment of this nerve.  The fascial hiatus also may serve as a potential tether in cases of inversion injuries causing injury to the superficial peroneal nerve.

 

REFERENCES: Mizel MS, Miller RA, Scioli MW (eds): Orthopaedic Knowledge Update: Foot and Ankle 2.  Rosemont, IL, American Academy of Orthopaedic Surgeons, 1998, pp 101-111.

Baumhauer JF: Superficial peroneal nerve entrapment.  Foot Ankle Clin 1998;3:439-446.

 

 

 

 

25.      A 35-year-old runner has pain beneath the second metatarsophalangeal joint.  He reports that he has significantly decreased his running distance since the onset of the pain.  He denies any history of trauma or injury to the foot.  A radiograph is shown in Figure 14.  Initial management should consist of

 

1-         resectional arthroplasty.

2-         metatarsophalangeal joint arthrodesis.

3-         dorsal closing wedge osteotomy.

4-         physical therapy.

5-         a rigid steel shank shoe with a rocker bottom sole.

 

PREFERRED RESPONSE: 5

 

DISCUSSION: The presence of the relatively long second metatarsal, along with the close approximation of the second and third metatarsal heads, are consistent with second metatarsophalangeal tenosynovitis.  The hallmark of initial management is conservative.  Modalities include taping, nonsteroidal anti-inflammatory drugs, metatarsal pads, and cortisone injections.  Trepman and Yeo combined the use of a cortisone injection with a rocker bottom sole.  Mizel and Michelson reported their results using an extended rigid steel shank shoe along with a cortisone injection.

 

REFERENCES: Trepman E, Yeo SJ: Nonoperative treatment of metatarsophalangeal joint synovitis.  Foot Ankle Int 1995;16:771-777.

Mizel MS, Michelson JD: Nonsurgical treatment of monarticular nontraumatic synovitis of the second metatarsophalangeal joint.  Foot Ankle Int 1997;18:424-426.

 

 

 

 

26.      Figures 15a and 15b show the radiographs of an 18-year-old mountain biker who came off of a 15-foot ramp and sustained an injury to his ankle.  Because the local rural hospital had no orthopaedic surgeon available, he was transported to a Level 1 emergency department 10 hours after his initial injury.  Examination reveals that the injury remains closed.  Management should consist of

 

1-         immediate CT to define the fragments.

2-         closed reduction and short leg casting, followed by delayed open reconstruction of the fracture in 2 weeks.

3-         immediate open reduction and internal fixation of the fracture.

4-         application of an external fixator with traction for provisional reduction and delayed open reconstruction.

5-         primary fusion of the ankle.

 

PREFERRED RESPONSE: 4

 

DISCUSSION: High-energy tibial pilon fractures involve disruption of the soft-tissue envelope with significant lower extremity edema.  Definitive reconstruction of the comminuted distal tibia should be delayed for at least 7 days to allow edema to dissipate, lowering the risk of skin necrosis.  An external fixator is the best method to keep the ankle at anatomic length while preventing skin necrosis. Ligamentotaxis will hold the fragments reduced to allow the edema to dissipate.  CT may be obtained in traction to localize the individual fragments and plan surgical incisions and subsequent fixation.  Short leg casting will not provide adequate ligamentotaxis to hold the fragments reduced and prevent skin compromise.  Primary fusion of the ankle in an unstable tibial pilon fracture is prone to a poor result from nonunion or malunion. 

 

REFERENCES: Tornetta P III, Weiner L, Bergman M, et al: Pilon fractures: Treatment with combined internal and external fixation.  J Orthop Trauma 1993;7:489-496.

Sirkin M, Sanders R, DiPasquale T, Herscovici D Jr: A staged protocol for soft tissue management in the treatment of complex pilon fractures.  J Orthop Trauma 1999;13:78-84.

 

 

 

 

27.      A 47-year-old woman has had medial ankle pain and swelling for the past 3 months.  She recalls no specific injury, and casting and nonsteroidal anti-inflammatory drugs have failed to provide relief.  Examination reveals a pes planus with heel valgus that is passively correctable.  Radiographs show no evidence of arthritis.  An MRI scan is shown in Figure 16.  What is the most appropriate surgical procedure to alleviate her pain?

 

1-         Triple arthrodesis

2-         Isolated flexor digitorum longus transfer

3-         Flexor digitorum longus transfer with medial displacement calcaneal osteotomy

4-         Debridement and direct repair of the posterior tibial tendon

5-         Repair of the spring ligament

 

PREFERRED RESPONSE: 3

 

DISCUSSION: The patient has a stage II posterior tibial tendon tear with a supple foot; therefore, the treatment of choice is flexor digitorum longus transfer with medial displacement calcaneal osteotomy.  Triple arthrodesis is not indicated, and isolated tendon transfer will stretch out in the face of persistent heel valgus.  Direct repair of the posterior tibial tendon or repair of the spring ligament is not sufficient to correct the deformity.

 

REFERENCES: Myerson MS, Corrigan J: Treatment of posterior tibial tendon dysfunction with flexor digitorum longus tendon transfer and calcaneal osteotomy.  Orthopedics 1996;19:383-388.

Mosier-LaClair S, Pomeroy G, Manoli A II: Operative treatment of the difficult stage 2 adult acquired flatfoot deformity.  Foot Ankle Clin 2001;6:95-119.

 

 

 

 

28.      A 38-year-old woman has a lesion on her left foot that has increased in size over the past 6 months.  The clinical photograph is shown in Figure 17a, and a photomicrograph of the biopsy specimen is shown in Figure 17b.  What is the most likely diagnosis?

 

1-         Kaposi sarcoma

2-         Squamous cell carcinoma

3-         Pigmented basal cell carcinoma

4-         Melanoma

5-         Basillary angiomatosis

 

PREFERRED RESPONSE: 4

 

DISCUSSION: Melanoma comprises 25% of lower extremity lesions and is the most common malignant tumor of the foot.  The preferred treatment is wide resection.

 

REFERENCES: Hughes LE, Horgan K, Taylor BA, Laidler P: Malignant melanoma of the hand and foot: Diagnosis and management.  Br J Surg 1985;72:811-815.

Fortin PT, Freiberg AA: Malignant melanoma of the foot and ankle.  J Bone Joint Surg Am 1995;77:1396-1403.

 

 

29.      A 16-year-old boy has had a painful ingrown nail on his great toe for the past 3 months.  When initial management consisting of soaking the foot in Epsom salts and trimming the nail failed to provide relief, his family physician recommended 2 weeks of oral antibiotics.  His symptoms persist, and he is now seeking a second opinion.  A clinical photograph is shown in Figure 18.  Management should now consist of

 

1-         povidone-iodine soaks and oral antibiotics.

2-         povidone-iodine soaks and IV antibiotics.

3-         partial nail plate removal.

4-         phenol nail matrix ablation.

5-         surgical nail matrix ablation.

 

PREFERRED RESPONSE: 3

 

DISCUSSION: The patient has a chronic ingrown nail on his great toe, which is not an uncommon occurrence in teenagers because of improper nail care.  There is local infection and a foreign body reaction because of the nail.  Continued conservative management with soaks and antibiotics will not improve the clinical situation.  In the presence of local chronic infection, nail matrix ablation is contraindicated.  Additionally, in the absence of a history of an ingrown nail, a nail matrix ablation is not medically indicated.  The appropriate treatment is partial removal of the nail plate.  With nail plate removal, the inflammation and local infection will resolve rapidly.

 

REFERENCES: Pettine KA, Cofield RH, Johnson KA, Bussey RM: Ingrown toenail: Results of surgical treatment.  Foot Ankle 1988;9:130-134.

Coughlin MJ, Mann RA: Toenail abnormalities, in Coughlin MJ, Mann RA (eds): Surgery of the Foot and Ankle, ed 7.  St Louis, MO, Mosby, 1999, vol 2, pp 1033-1070.

 

 

 

 

30.      A 28-year-old man has a painful nodule on the plantar aspect of his foot in the midarch.  Use of a soft orthosis has failed to provide relief.  Examination reveals that the mass is approximately 2 1/2 cm in diameter, firm, and tender to palpation. An MRI scan confirms the presence of a plantar fibroma.  Management should now consist of

 

1-         local resection of the fibroma.

2-         resection of the entire plantar fascia.

3-         local radiation therapy.

4-         resection and adjuvant chemotherapy.

5-         Syme amputation.

 

PREFERRED RESPONSE: 2

 

DISCUSSION: Plantar fibromas have an extremely high recurrence rate (approximately 60%) with local excision only.  Resection of the entire plantar fascia is effective at irradicating the lesion.  There is no role for chemotherapy or amputation with plantar fibromatosis.  Radiation therapy may be helpful in combination with resection of the plantar fascia.

 

REFERENCES: Kirby EJ, Shereff MJ, Lewis MM: Soft-tissue tumors and tumor-like lesions of the foot: An analysis of 83 cases.  J Bone Joint Surg Am 1989;71:621-626.

Cavolo DJ, Sherwood GF: Dupuytren’s disease of the plantar fascia.  J Foot Surg 1982;21:12-15.

 

 

 

 

31.      A child born with myelomeningocele is expected to be an ambulator with bracing.  Examination by the consulting orthopaedic surgeon reveals rigid clubfeet in addition to the neurologic issues.  Management should consist of

 

1-         immediate casting with the expectation of a satisfactory correction.

2-         immediate casting with the expectation that surgical correction will be needed.

3-         immediate surgery to correct the deformity.

4-         delayed casting and corrective bracing.

5-         therapeutic and frequent physical therapy to stretch the soft tissues and observe the skin.

 

PREFERRED RESPONSE: 2

 

DISCUSSION: In a child with myelomeningocele, the guiding principle of treatment is to achieve a plantigrade foot by the time the child is ready to stand.  The standard clubfoot protocol should be followed, but these children will require an aggressive surgical release to obtain a sufficient correction.

 

REFERENCES: Mizel MS, Miller RA, Scioli MW (eds): Orthopaedic Knowledge Update: Foot and Ankle 2.  Rosemont, IL, American Academy of Orthopaedic Surgeons, 1998, pp 65-78.

Kasser JE (ed): Orthopaedic Knowledge Update 5.  Rosemont, IL, American Academy of Orthopaedic Surgeons, 1996, pp 503-514.

 

 

 

 

32.      A 42-year-old man has a symptomatic flatfoot deformity and walks with a slight limp after falling off a scaffold 9 months ago.  He also reports that he has had difficulty returning to work.  Orthotics have failed to provide relief.  Current radiographs are shown in Figures 19a and 19b.  To relieve his pain and return the patient to work, treatment should consist of

 

1-         lateral column lengthening.

2-         open reduction and internal fixation.

3-         double arthrodesis (talonavicular and calcaneocuboid joints).

4-         tarsometatarsal arthrodesis.

5-         medial displacement calcaneal osteotomy and tendon transfer.

 

PREFERRED RESPONSE: 4

 

DISCUSSION: Because the patient has sustained a tarsometatarsal injury with midfoot sag, the treatment of choice is a tarsometatarsal arthrodesis.  The cause of his flatfoot deformity is secondary to the tarsometatarsal injury and not from posterior tibialis tendon deficiency.  Lateral column lengthening, double arthrodesis, and calcaneal osteotomy are not indicated.  Although open reduction and internal fixation may be performed late when arthritis is present, these procedures are less likely to succeed.

 

REFERENCES: Komenda GA, Myerson MS, Biddinger KR: Results of arthrodesis of the tarsometatarsal joints after traumatic injury.  J Bone Joint Surg Am 1996;78:1665-1676.

Sangeorzan BJ, Veith RG, Hansen ST Jr: Salvage of Lisfranc’s tarsometatarsal joint by arthrodesis.  Foot Ankle 1990;10:193-200.

 

 

 

 

33.      A 16-year-old boy has a symptomatic flatfoot deformity that is causing pain, skin breakdown, and shoe wear problems.  Shoe modification and an orthosis have failed to provide relief.  Examination reveals hindfoot valgus, talonavicular sag, and forefoot abduction that are all passively correctable.  Treatment should consist of

 

1-         medial soft-tissue tightening.

2-         medial soft-tissue tightening with lateral column lengthening.

3-         medial soft-tissue tightening with talonavicular fusion.

4-         medial displacement osteotomy with flexor digitorum longus transfer into the tarsal navicular.

5-         triple arthrodesis with lateral column lengthening.

 

PREFERRED RESPONSE: 2

 

DISCUSSION: The patient has a supple planovalgus deformity that is passively fully correctable, and nonsurgical management has failed to provide relief.  Lateral column lengthening with medial soft-tissue tightening will correct the deformity and maintain a flexible foot.  Arthrodesis is not recommended for a supple, correctable deformity because of loss of motion and long-term degeneration of surrounding joints.  Medial displacement calcaneal osteotomy is generally reserved for an adult-acquired flexible flatfoot.

 

REFERENCES: Beaty JH (ed): Orthopaedic Knowledge Update 6.  Rosemont, IL, American Academy of Orthopaedic Surgeons, 1999, pp 613-631.

Evans D: Calcaneo-valgus deformity.  J Bone Joint Surg Br 1975;57:270-278.

Mosca VS: Calcaneal lengthening for valgus deformity of the hindfoot: Results in children who had severe, symptomatic flatfoot and skewfoot.  J Bone Joint Surg Am 1995;77:500-512.

 

 

 

 

34.      A 48-year-old man has had pain and swelling of the hallux metatarsophalangeal joint for the past 9 months.  A rocker bottom stiff-soled shoe has failed to provide relief; however, two cortisone injections have temporarily alleviated his symptoms.  The radiographs shown in Figures 20a and 20b reveal diffuse arthritis of the entire hallux metatarsophalangeal joint.  What is the most definitive surgical treatment? 

 

1-         Dorsal cheilectomy

2-         Keller resection arthroplasty

3-         Silastic joint replacement

4-         Hallux metatarsophalangeal arthrodesis

5-         Hallux valgus correction

 

PREFERRED RESPONSE: 4

 

DISCUSSION: Because the radiographs demonstrate severe arthritis, hallux metatarsophalangeal arthrodesis is the treatment of choice.  Cheilectomy alone will not relieve pain because the entire joint is degenerative.  Joint replacement has not been shown to be a long-term solution.  Keller resection arthroplasty is not indicated in younger active patients.  Hallux valgus correction will not address arthritis of the joint and could stiffen the joint further.

 

REFERENCES: Smith RW, Joanis TL, Maxwell PD: Great toe metatarsophalangeal joint arthrodesis: A user-friendly technique.  Foot Ankle 1992;13:367-377.

Mann RA: Hallux rigidus.  Instr Course Lect 1990;39:15-21.

 

 

 

35.      During reconstruction of insertional gaps of a chronic Achilles tendon rupture, what tendon provides the most direct route of transfer?

 

1-         Flexor hallucis longus

2-         Flexor digitorum longus

3-         Peroneus longus

4-         Posterior tibial

5-         Quadratus planti

 

PREFERRED RESPONSE: 1

 

DISCUSSION: The flexor hallucis longus tendon provides the best, most direct route of transfer for filling Achilles tendon gaps.  The tendon lies lateral to the neurovascular structures, making it safe for harvest and providing a direct route for transfer into the calcaneus without crossing these important structures.  The flexor hallucis longus tendon also has muscle belly that extends distal on the tendon itself, often beyond the actual tibiotalar joint.  When the tendon is transferred, this muscle belly brings excellent blood supply to the anterior portion of the reconstruction.

 

REFERENCES: Wilcox DK, Bohay DR, Anderson JG: Treatment of chronic achilles tendon disorders with flexor hallucis longus tendon transfer/augmentation.  Foot Ankle Int 2000;21:1004-1010.

Wapner K, Pavlock GS, Hecht PJ, Naselli F, Walther R: Repair of chronic Achilles tendon rupture with flexor hallucis longus tendon transfer.  Foot Ankle Int 1993;14:443-449.

 

36.      A 27-year-old woman with Down syndrome has a severe bunion with pain and deformity in the left forefoot.  Nonsurgical management has failed to provide relief.  She does not use any assistive ambulatory devices.  A radiograph is shown in Figure 21.  Treatment should now consist of

 

1-         distal chevron bunionectomy.

2-         proximal first metatarsal osteotomy with distal soft-tissue realignment.

3-         double osteotomy of the first metatarsal with distal soft-tissue realignment.

4-         arthrodesis of the first metatarsophalangeal joint.

5-         Keller bunionectomy.

 

PREFERRED RESPONSE: 4

 

DISCUSSION: The patient requires an arthrodesis of the first metatarsophalangeal joint because of the abnormal neuromuscular forces.  The more traditional bunionectomies such as a distal chevron bunionectomy, a proximal first metatarsal osteotomy, and a double osteotomy have a high failure rate because of the underlying Down syndrome.  The Keller procedure is indicated for older, sedentary individuals and has little role in the management of a neuromuscular bunion.

 

REFERENCES: Coughlin MJ, Abdo RV: Arthodesis of the first metatarsophalangeal joint with Vitallium plate fixation.  Foot Ankle Int 1994;15:18-28.

Mann RA: Disorders of the first metatarsophalangeal joint.  J Am Acad Orthop Surg 1995;3:34-43.

 

 

 

 

37.      Which of the following is considered the most important factor in eliminating infection in chronic osteomyelitis?

 

1-         Antibiotic use

2-         Complete debridement of the compromised bone and soft tissue

3-         Immobilization of the infected joint

4-         Irrigation of the tissues

5-         Arthrodesis of the adjacent joint

 

PREFERRED RESPONSE: 2

 

DISCUSSION: The most important factor in eliminating infection in chronic osteomyelitis is a complete debridement of the compromised bone and soft tissue.  Antibiotics should be used in conjunction with surgical debridement.  However, the foundation of treating infected bone is removal of the diseased tissue.

 

REFERENCES: Cierny G III, Cook WG, Mader JT: Ankle arthrodesis in the presence of ongoing sepsis: Indications, methods, and results.  Orthop Clin North Am 1989;20:709-721.

Cierny G, Zorn EZ: Arthrodesis of the tibiotalar joint for sepsis.  Foot Ankle Clin 1996;1:177-197.

Richter D, Hahn MP, Laun RA, Ekkernkamp A, Muhr G, Osterman PA: Arthrodesis of the infected ankle and subtalar joint: Technique, indications and results of 45 consecutive cases.  J Trauma 1999;47:1072-1078.

 

 

 

 

38.      A 35-year-old woman reports worsening pain after undergoing a neurectomy in the third interspace for a Morton’s neuroma 12 months ago.  She states that the pain is sharp and electrical, worse than before her surgery, and prevents her from participating in her usual work and exercise activities.  Use of wider shoes and pads used before her surgery have failed to provide relief.  Examination does not reveal any deformity or inflammation.  Tenderness along with neuritic pain occurs with compression of the plantar aspect of the foot between the third and fourth metatarsal head area.  To most reliably alleviate her pain, management should consist of

 

1-         serial injection of steroids into the area of discomfort.

2-         custom orthotics with a second metatarsal pad.

3-         physical therapy for transcutaneous electrical nerve stimulation and desensitization.

4-         plantar condylectomy of the second and third metatarsal heads.

5-         plantar exploration and revision neurectomy to a more proximal level.

 

PREFERRED RESPONSE: 5

 

DISCUSSION: Most patients with a significant recurrent neuroma will not obtain relief with conservative methods.  Pain results from a stump neuroma at the weight-bearing area from too short of a resection of the nerve or from regrowth of the remaining nerve end.  Although steroid injection may be helpful in localizing symptoms or providing temporary relief, it rarely cures a stump neuroma.  Orthotics with a metatarsal pad will likely increase pressure and pain at the neuroma site.  Physical therapy could temporize the symptoms but will not address the underlying problem.  Similarly, bone decompression alone will not alter the location of the neuroma stump.  Revision of the nerve to a more proximal level off of the weight-bearing area is the most likely method to succeed.  A plantar approach facilitates identification and ability to revise the nerve to a more proximal level.

 

REFERENCES: Mizel MS, Miller RA, Scioli MW (eds): Orthopaedic Knowledge Update: Foot and Ankle 2.  Rosemont, IL, American Academy of Orthopaedic Surgeons, 1998, pp 101-111.

Johnson JE, Johnson KA, Unni KK: Persistent pain after excision of an interdigital neuroma: Results of reoperation.  J Bone Joint Surg Am 1988;70:651-657.

Beskin JL, Baxter DE: Recurrent pain following interdigital neurectomy: A plantar approach.  Foot Ankle 1988;9:34-39.

Amis JA, Siverhus SW, Liwnicz BH: An anatomic basis for recurrence after Morton’s neuroma excision.  Foot Ankle 1992;13:153-156.

 

39.      What type of brace is shown in Figures 22a and 22b?

 

1-         Charcot restraining orthotic walker (CROW)

2-         University of California Biomechanics Laboratory (UCBL) orthosis

3-         Double upright ankle-foot orthosis

4-         Chopart’s prosthesis

5-         Below-knee prosthesis

 

PREFERRED RESPONSE: 1

 

DISCUSSION: The figures show a Charcot restraining orthotic walker (CROW).  This brace has been used as a customized total contact fit removable brace to maintain foot alignment as the patient evolves from Eichenholz stage 1 to Eichenholz stage 3 Charcot arthropathy.

 

REFERENCES: Mehta JA, Brown C, Sargeant N: Charcot restraint orthotic walker.  Foot Ankle Int 1998;19:619-623. 

Morgan JM, Biehl WC III, Wagner FW Jr: Management of neuropathic arthropathy with the Charcot restraint orthotic walker.  Clin Orthop 1993;296:58-63. 

 

 

 

 

40.      A 23-year-old man has pain and a callus beneath the second metatarsal head.  Initial management should consist of

 

1-         corn pads impregnated with salicylic acid.

2-         paring of the callus and an offloading orthosis.

3-         metatarsal dorsiflexion osteotomy.

4-         metatarsal head excision.

5-         plantar condylectomy.

 

PREFERRED RESPONSE: 2

 

DISCUSSION: The initial treatment of metatarsalgia with or without the presence of an intractable keratosis should be conservative.  Simple paring of the callus with elevation of the metatarsals may suffice.  A prefabricated “off-the-shelf” orthosis or felt pad can be used before investing in a custom orthosis.  The use of medicated pads can lead to greater amounts of keratosis and should be avoided.

 

REFERENCES: Mizel MS, Miller RA, Scioli MW (eds): Orthopaedic Knowledge Update: Foot and Ankle 2.  Rosemont, IL, American Academy of Orthopaedic Surgeons, 1998, pp 163-173.

Mizel MS, Yodlowski ML: Disorders of the lesser metatarsophalangeal joints.  J Am Acad Orthop Surg 1995;3:166-173.

 

 

 

41.       Figures 23a and 23b show the radiograph and clinical photograph of a patient who reports a reduced ability to flex the interphalangeal joint of her great toe after undergoing a Chevron-Akin bunionectomy.  What is the most likely cause?

 

1-         Interphalangeal joint arthritis

2-         Laceration of the flexor hallucis longus tendon

3-         Sesamoid entrapment

4-         Metatarsal malunion

5-         Phalangeal malunion

 

PREFERRED RESPONSE: 2

 

DISCUSSION: The flexor hallucis longus tendon is at risk during a Chevron-Akin osteotomy because of its close relationship to the base of the proximal phalanx.  The radiograph reveals a reduced ability to flex the interphalangeal joint secondary to the flexor hallucis longus laceration.  The other complications are not supported by the radiograph.

 

REFERENCES: Tollison ME, Baxter DE: Combination chevron plus Akin osteotomy for hallux valgus: Should age be a limiting factor?  Foot Ankle Int 1997;18:477-481.

Scaduto AA, Cracchiolo A III: Lacerations and ruptures of the flexor or extensor hallucis longus tendons.  Foot Ankle Clin 2000;5:725-736.

 

42.      A 45-year-old man has persistent hindfoot pain that is aggravated by weight-bearing activities.  History reveals that he sustained a calcaneus fracture 2 years ago, and he underwent a subtalar fusion 1 year ago.  Examination reveals tenderness in the sinus tarsi and across the transverse tarsal joint.  A plain radiograph and a CT scan are shown in Figures 24a and 24b.  A technetium Tc 99m bone scan reveals uptake at the subtalar joint and at the transverse tarsal joints.  Management should now consist of

 

1-         casting with an external bone stimulator.

2-         ankle arthrodesis.

3-         revision subtalar arthrodesis.

4-         conversion to triple arthrodesis without revision of the subtalar arthrodesis.

5-         conversion to triple arthrodesis with revision of the subtalar arthrodesis.

 

PREFERRED RESPONSE: 5

 

DISCUSSION: The patient has a nonunion at the subtalar joint because of poor preparation of the arthrodesis site with incomplete removal of the articular cartilage.  Clinically, he has arthritis at the transverse tarsal joint.  Casting with a bone stimulator is not expected to result in a union of the subtalar arthrodesis.  To address both the subtalar nonunion and the transverse tarsal joint arthritis, revision of the subtalar arthrodesis and conversion to a triple arthrodesis is the preferred option.

 

REFERENCES: Graves SC, Mann RA, Graves KO: Triple arthrodesis in older adults: Results after long-term follow-up.  J Bone Joint Surg Am 1993;75:355-362.

Haddad SL, Myerson MS, Pell RF IV, Schon LC: Clinical and radiographic outcome of revision surgery for failed triple arthrodesis.  Foot Ankle Int 1997;18:489-499.

Sangeorzan BJ, Smith D, Veith R, Hansen ST Jr: Triple arthrodesis using internal fixation in treatment of adult foot disorders.  Clin Orthop 1993;294:299-307. 

Sangeorzan BJ: Salvage procedures for calcaneus fractures.  Instr Course Lect 1997;46:339-346. 

Wapner KL: Triple arthrodesis in adults.  J Am Acad Orthop Surg 1998;6:188-196.

 

 

 

 

43.      A 21-year-old college student reports hearing a pop and has acute pain laterally over the ankle after twisting it during a recreational basketball game.  Examination 1 hour after the injury reveals minimal swelling and ecchymosis.  The anterior drawer sign is positive.  Radiographs reveal no evidence of a fracture.  What is the best course of action?

 

1-         Stress radiographs to evaluate the ankle ligaments

2-         MRI to evaluate possible ligamentous injury

3-         CT to evaluate possible osteochondral injury

4-         Functional brace treatment

5-         Surgical repair of the ankle ligaments

 

PREFERRED RESPONSE: 4

 

DISCUSSION: Even though the patient has a grade 3 ankle ligament injury, studies have shown that 95% of patients with a grade 3 injury that may include a complete tear of the ligaments will heal successfully with conservative functional management.  Extensive diagnostic evaluation with stress radiographs, CT, and MRI is not indicated.  Surgical reconstruction is not indicated because of the overwhelming success of conservative management; however, in the few patients where late instability develops, surgical reconstruction offers an excellent outcome.

 

REFERENCES: Carne P: Nonsurgical treatment of ankle sprains using the modified Sarmiento brace.  Am J Sports Med 1989;17:253-257.

Hamilton WG: Sprained ankles in ballet dancers.  Foot Ankle 1982;3:99-102.

 

 

 

 

44.      What significant structure is most at risk during a posterior approach of the Achilles tendon near its musculotendinous junction?

 

1-         Saphenous vein

2-         Saphenous nerve

3-         Posterior tibial nerve

4-         Sural nerve

5-         Plantaris muscle

 

PREFERRED RESPONSE: 4

 

DISCUSSION: The sural nerve crosses near the midline at the level of the musculotendinous junction before descending to its more lateral location distally.  The saphenous nerve and vein are further medial and at less risk.  The posterior tibial nerve is at risk only during deep dissection, such as harvesting flexor hallucis longus tendon graft.  The plantaris muscle lies in this area but is of little clinical significance.

 

REFERENCES: Webb J, Moorjani N, Radford M: Anatomy of the sural nerve and its relation to the Achilles tendon.  Foot Ankle Int 2000;21:475-477.

Mizel MS, Miller RA, Scioli MW (eds): Orthopaedic Knowledge Update: Foot and Ankle 2.  Rosemont, IL, American Academy of Orthopaedic Surgeons, 1998, pp 101-111.

 

 

 

 

45.      Figure 25 shows the clinical photograph of a 48-year-old man who has had a forefoot ulcer for the past 4 months.  History reveals that he has had type II diabetes mellitus for the past 10 years.  Examination reveals sensory and motor neuropathy, with weak ankle dorsiflexion.  The ankle cannot be passively dorsiflexed past a neutral position.  Initial management should consist of

 

1-         an extra-depth shoe with an accommodative orthosis.

2-         surgical debridement and metatarsal head resection.

3-         posterior tibial tendon transfer through the interosseous membrane.

4-         Achilles tendon lengthening and total contact casting.

5-         hyperbaric oxygen therapy.

 

PREFERRED RESPONSE: 4

 

DISCUSSION: Foot deformity and decreased joint motion have been associated with increased plantar pressures and an increased risk of ulceration.  In a partial-thickness ulcer without exposed bone or tendon, total contact casting is highly effective.  Concomitant Achilles tendon lengthening increases the likelihood that healing of the ulcer can be obtained and perhaps more importantly, maintained.

 

REFERENCES: Lin SS, Lee TH, Wapner KL: Plantar forefoot ulceration with equinus deformity of the ankle in diabetic patients: The effect of tendo-Achilles lengthening and total contact casting.  Orthopedics 1996;19:465-475.

Armstrong DG, Stacpoole-Shea S, Nguyen H, Harkless LB: Lengthening of the Achilles tendon in diabetic patients who are at high risk for ulceration of the foot.  J Bone Joint Surg Am 1999;81:535-538.

 

 

 

 

46.      An active 36-year-old woman with rheumatoid arthritis has continued forefoot discomfort despite the use of orthotics and shoe wear modifications.  A radiograph and a clinical photograph are shown in Figures 26a and 26b.  Treatment at this point should consist of

 

1-         hallux valgus correction and lesser metatarsophalangeal joint synovectomies.

2-         hallux valgus correction and lesser metatarsal head resections.

3-         first metatarsophalangeal joint fusion and lesser metatarsal head resections.

4-         first metatarsophalangeal joint fusion and lesser metatarsophalangeal joint synovectomies.

5-         Keller resection arthroplasty and lesser metatarsal head resections.

 

PREFERRED RESPONSE: 3

 

DISCUSSION: In a patient with inflammatory arthritis, advanced hallux valgus deformity in conjunction with lesser metatarsophalangeal joint destruction and subluxation warrants fusion of the first metatarsophalangeal joint and lesser metatarsal head resections.  Hallux valgus correction will fail because of incompetent soft tissues.  A Keller resection arthroplasty is not indicated in this age group.  Synovectomy is contraindicated because of evidence of erosive changes of the lesser metatarsophalangeal joints.

 

REFERENCES: Ouzounian T: Rheumatoid arthritis of the foot & ankle, in Myerson MS (ed): Foot & Ankle Disorders.  Philadelphia, PA, WB Saunders, 2000, vol 2, pp 1189-1204. 

Mann RA, Thompson FM: Arthrodesis of the first metatarsophalangeal joint for hallux valgus in rheumatoid arthritis.  J Bone Joint Surg Am 1984;66:687-692. 

Coughlin MJ: Rheumatoid forefoot reconstruction: A long-term followup study.  J Bone Joint Surg Am 2000;82:322-341.  

 

 

 

47.      The Keller proximal phalanx resection procedure is most useful for which of the following conditions?

 

1-         Mild bunion deformity in a 35-year-old woman

2-         Severe bunion deformity in a patient with rheumatoid arthritis

3-         Recurrent juvenile bunion deformity

4-         Plantar neuropathic ulcer of the great toe

5-         Bunion deformity associated with a hypermobile first tarsometatarsal joint

 

PREFERRED RESPONSE: 4

 

DISCUSSION: A Keller proximal phalanx resection procedure usually results in reduced weight bearing under the first ray because of shortening of the toe and disruption of intrinsic flexor function.  This can be an effective method of offloading a neuropathic ulcer under the great toe at the interphalangeal or metatarsophalangeal joint area.  However, these features are generally undesirable in young active patients.  The procedure has a high rate of recurrent deformity in patients with rheumatoid arthritis.  It would exacerbate transfer metatarsalgia in a patient with a hypermobile first ray.

 

REFERENCES: Lin SS, Bono CM, Lee TH: Total contact casting and Keller arthroplasty for diabetic great toe ulceration under the interphalangeal joint.  Foot Ankle Int 2000;21:588-593.

Downs DM, Jacobs RL: Treatment of resistant ulcers on the plantar surface of the great toe in diabetics.  J Bone Joint Surg Am 1982;64:930-933.

 

 

 

48.      An active 60-year-old man is evaluated 4 years following surgical correction of a hallux valgus deformity.  The patient reports that a hallux varus deformity developed rapidly following his initial surgery.  Conservative management consisting of wider shoes, toe strapping, and anti-inflammatory drugs has failed to provide relief.  Examination reveals a hallux varus deformity with restricted painful motion of the metatarsophalangeal joint and callus formation under the second metatarsal head.  What is the next most appropriate step in management?

 

1-         Fascial arthroplasty

2-         Metatarsophalangeal joint arthrodesis

3-         Metatarsophalangeal joint Silastic arthroplasty

4-         Extensor tendon reconstruction

5-         Keller resection arthroplasty

 

PREFERRED RESPONSE: 2

 

DISCUSSION: Hallux varus may occur as a complication following hallux valgus surgery, most commonly a modified McBride-type procedure.  Conservative management is the initial treatment of choice; however, if unsuccessful, surgical options for reconstruction include soft-tissue reconstruction or metatarsophalangeal joint arthrodesis.  The patient has evidence of joint arthrosis, making an arthrodesis the preferred method of reconstruction.  Fascial arthroplasty, Silastic arthroplasty, and Keller resection arthroplasty will not correct the underlying deformity.

 

REFERENCES: Kitaoka HB, Patzer GL: Arthrodesis versus resection arthroplasty for failed hallux valgus operations.  Clin Orthop 1998;347:208-214.

Ouzounian TJ: Metatarsophalangeal arthrodesis for salvage of failed hallux valgus surgery.  Foot Ankle Clin 1997;2:741-752.

 

 

 

49.      A newborn has been referred for evaluation of a deformed foot.  Prenatal and birth history are unremarkable.  Examination reveals a rocker bottom appearance to the foot, and a longitudinal arch cannot be created.  A palpable lump is appreciated on the plantar medial surface.  What is the best course of action?

 

1-         Parental education and reassurance

2-         Serial casting of the forefoot

3-         Complete peritalar release

4-         Lengthening of the Achilles tendon and soft-tissue balancing

5-         Reverse last shoes

 

PREFERRED RESPONSE: 2

 

DISCUSSION: The patient has congenital vertical talus.  The navicular is irreducibly dorsally dislocated on the talus with the talar head prominent on the plantar medial aspect of the foot.  Initial management involves corrective casting for 3 months to stretch the dorsal tendons, skin, and neurovascular structures.  Surgical reconstruction is often needed and is indicated when the patient is age 6 to 12 months.  Reconstruction requires both bony and soft-tissue procedures.

 

REFERENCES: Napiontek M: Congenital vertical talus: A retrospective and critical review of 32 feet operated on by peritalar reduction.  J Pediatr Orthop 1995;4:179-187.

Oppenheim W, Smith C, Christie W: Congenital vertical talus.  Foot Ankle 1985;5:198-204.

 

 

 

 

50.      Which of the following is considered an inherent problem in using the distal oblique shortening (Weil) metatarsal osteotomy for dorsal metatarsophalangeal subluxation?

 

1-         Nonunion of the osteotomy

2-         Malunion of the osteotomy

3-         Excessive shortening that results in transfer lesions to the adjacent toes

4-         Plantar translation that results in metatarsalgia

5-         Dorsal positioning of the intrinsic tendons and recurrent dorsal contracture of the toe

 

PREFERRED RESPONSE: 5

 

DISCUSSION: The distal oblique shortening (Weil) metatarsal osteotomy has not been associated with transfer lesions to the extent of other shortening osteotomies, and malunions and nonunions are unusual complications.  Recurrent dorsal contracture of the toe has been reported.  Recommendations to reduce this problem include release of the dorsal capsule and tendons, as well as a flexor tendon transfer.  A potential cause suspected for this phenomenon is the relatively dorsal positioning of the intrinsic tendons after plantar displacement of the metatarsal head.

 

REFERENCES: Trnka HJ, Nyska M, Parks BG, Myerson MS: Dorsiflexion contracture after the Weil osteotomy: Results of cadaver study and three-dimensional analysis.  Foot Ankle Int 2001;22:47-50.

Trnka HJ, Muhlbauer M, Zettl R, Myerson MS, Ritschl P: Comparison of the results of the Weil and Helal osteotomies for the treatment of metatarsalgia secondary to dislocation of the lesser metatarsophalangeal joints.  Foot Ankle Int 1999;20:72-79.

Vandeputte G, Dereymaeker G, Steenwerckx A, Peeraer L: The Weil osteotomy of the lesser metatarsals: A clinical and pedobarographic follow-up study.  Foot Ankle Int 2000;21:370-374.

 

 

 

51.       A 47-year-old woman underwent a distal chevron bunionectomy 2 months ago.  Her postoperative recovery had been uneventful until 1 week ago.  She now has new onset pain and dorsal swelling in the area of the third metatarsal.  A radiograph is shown in Figure 27.  What is the most likely diagnosis?

 

1-         Transfer metatarsalgia

2-         Morton’s neuroma 

3-         Metatarsal stress fracture

4-         Freiberg’s infraction

5-         Metatarsophalangeal synovitis

 

PREFERRED RESPONSE: 3

 

DISCUSSION: Based on findings of a sudden increase in pain with associated swelling, the most likely diagnosis is a stress fracture.  The initial radiographic findings usually will be negative.  Morton’s neuroma and transfer metatarsalgia are not associated with swelling.  Metatarsophalangeal synovitis usually involves the second metatarsophalangeal joint.  Freiberg’s infraction is seen clearly on a radiograph.

 

REFERENCE: Boden BP, Osbahr DC: High risk stress fractures: Evaluation and treatment.  J Am Acad Orthop Surg 2000;8:344-353.

 

 

 

 

52.      A 32-year-old runner has pain in the medial arch that radiates into the medial three toes.  He reports the presence of pain only when running.  Examination reveals normal hindfoot alignment.  There is a weakly positive Tinel’s sign over the posterior tibial nerve.  Tenderness is noted with palpation over the plantar medial area in the vicinity of the navicular tuberosity.  What is the most likely diagnosis?

 

1-         Plantar fasciitis

2-         Sinus tarsi syndrome

3-         Medial plantar nerve entrapment

4-         Entrapment of the nerve to the abductor digiti quinti

5-         Anterior tibial tendinitis

 

PREFERRED RESPONSE: 3

 

DISCUSSION: The examination findings reveal that there is specific involvement of the medial plantar nerve by the distribution of the pain medially.  The symptoms exclude the possibility of plantar fasciitis and anterior tibial tendinitis.  Sinus tarsi syndrome would produce anterolateral symptoms rather than medial symptoms.

 

REFERENCES: Rask MR: Medial plantar neurapraxia (jogger’s foot): Report of three cases.  Clin Orthop 1978;134:193-195. 

Murphy PC, Baxter DE: Nerve entrapment of the foot and ankle in runners.  Clin Sports Med 1985;4:753-763. 

Lutter LD: Surgical decisions in athletes’ subcalcaneal pain.  Am J Sports Med 1986;14:481-485.

 

 

 

 

53.      A 58-year-old woman with rheumatoid arthritis and a severe hindfoot valgus deformity now reports recurrent lateral ankle pain.  Examination reveals pain over the fibula and sinus tarsi, with a valgus hindfoot that is passively correctable.  Despite the use of an ankle-foot orthosis, this is the second time this problem has occurred.  Radiographs and a clinical photograph are shown in Figures 28a through 28c.  What is the next most appropriate step in treatment?

 

1-         Intramedullary screw fixation of the fibula

2-         Plating of the fibula with a one third tubular plate

3-         Subtalar arthrodesis with deformity correction

4-         Varus producing distal tibial osteotomy

5-         Ankle arthrodesis

 

PREFERRED RESPONSE: 3

 

DISCUSSION: Excessive hindfoot valgus can lead to abutment between the calcaneus and fibula.  This valgus force can lead to a stress fracture of the distal fibula.  Surgery may be required if an insufficiency fracture recurs despite orthotic management.  Of the choices listed, a subtalar arthrodesis is most likely to achieve rebalancing of the foot at the level of the deformity.

 

REFERENCES: Stephens HM, Walling AK, Solmen JD, Tankson CJ: Subtalar repositional arthrodesis for adult acquired flatfoot.  Clin Orthop 1999;365:69-73

Easley ME, Trnka HJ, Schon LC, Myerson MS: Isolated subtalar arthrodesis. J Bone Joint Surg Am 2000;82:613-624.

 

 

 

 

54.      A 7-year-old girl reports foot pain and has difficulty ambulating.  History reveals that she fell off a scooter 1 week ago, and there is possible exposure to a tick bite.  A radiograph is shown in Figure 29.  What is the best course of action?

 

1-         Consultation with an infectious disease specialist

2-         Symptomatic treatment, with a full return to activities in 4 to 6 weeks

3-         Cast immobilization for 6 to 12 weeks with the expectation of full recovery

4-         A rheumatologic work-up with the expectation of significant long-term sequelae

5-         Bone biopsy

 

PREFERRED RESPONSE: 3

 

DISCUSSION: The child has Kohler’s disease.  This is a self-limiting osteochondritis of the navicular.  It is treated symptomatically with initial cast immobilization for 6 to 12 weeks, followed possibly by orthotic management.  Findings shown in the radiograph usually will normalize within 1 year, and there are no long-term sequelae.

 

REFERENCES: Borges JL, Guille JT, Bowen JR: Kohler’s bone disease of the tarsal navicular.  J Pediatr Orthop 1995;15:596-598.

Mizel MS,  Miller RA, Scioli MW (eds): Orthopaedic Knowledge Update: Foot and Ankle 2.  Rosemont, IL, American Academy of Orthopaedic Surgeons, 1998, pp 65-78.

 

 

 

55.      A 45-year-old man has severe pain in both feet after his boots become wet while hunting.  Examination 3 hours after the onset of symptoms reveals that his feet are cold to touch and the skin appears blanched.  Management should consist of

 

1-         slow rewarming in cool 77 degrees F (25 degrees C) water.

2-         rapid rewarming in a footbath at 104.0 degrees F to 107.6 degrees F
(40 degrees C to 42 degrees C).

3-         rewarming in 98.6 degrees F (37 degrees C) water.

4-         heated blankets at 100.4 degrees F (38 degrees C).

5-         a heating pad at 104.0 degrees F (40 degrees C).

 

PREFERRED RESPONSE: 2

 

DISCUSSION: The patient has frostbite involving both feet.  Rapid rewarming in a protected environment is the initial treatment.  A footbath with water at 104.0 degrees F to 107.6 degrees F (40 degrees C to 42 degrees C) is ideal.  This facilitates a uniform rewarming of the involved tissue.  The other choices are less than ideal.  Appliances such as heating pads provide uneven heating and may actually burn the skin.

 

REFERENCES: Pinzur MS: Frostbite: Prevention and treatment.  Biomechanics 1997;4:14-21.

Fritz RL, Perrin DH: Cold exposure injuries: Prevention and treatment.  Clin Sports Med 1989;8:111-128.

 

 

 

56.      An 83-year-old woman with diabetes mellitus has a history of recurrent infection over the medial aspect of her great toe and has had a painless bunion for the past 45 years.  Shoe wear modifications have failed to provide relief.  Pedal pulses are palpable.  Figures 30a and 30b show the clinical photograph and radiograph.  Management should now consist of

 

1-         observation.

2-         first metatarsal head excision.

3-         simple bunionectomy (medial exostectomy and capsular repair).

4-         bunionectomy with first metatarsophalangeal fusion.

5-         distal soft-tissue procedure with proximal osteotomy.

 

PREFERRED RESPONSE: 3

 

DISCUSSION: The presence of recurrent breakdown over the medial eminence despite shoe wear modifications is an indication for surgery.  A number of factors must be considered when deciding on an appropriate course of treatment.  These include age, activity level, joint congruency, joint degeneration, and the patient’s symptoms and expectations.  The indications for a simple bunionectomy are rather limited.  In this patient, the goal of surgery is to alleviate the recurrent infection by removal of a large medial eminence.  Because the bunion is painless and long-standing, it does not warrant treatment.

 

REFERENCES: Mizel MS, Miller RA, Scioli MW (eds): Orthopaedic Knowledge Update:  Foot and Ankle 2.  Rosemont, IL, American Academy of Orthopaedic Surgeons, 1998, pp 123-134.

Abidi NA, Conti SF: The clinical and radiographic anatomy of hallux valgus and surgical algorithm.  Foot Ankle Clin 1997;2:599-626.

 

 

 

 

57.      Varus deformity after talar fractures is often seen due to collapse of the medial cortex.  What artery supplies this portion of the talus?

 

1-         Artery of the tarsal sinus

2-         Artery of the tarsal canal

3-         Peroneal

4-         Perforating branch of the dorsalis pedis artery

5-         Lateral malleolar

 

PREFERRED RESPONSE: 2

 

DISCUSSION: The artery of the tarsal canal is a branch of the posterior tibial artery.  Among the branches of the artery of the tarsal canal is the deltoid artery.  This arterial complex supplies the medial one third of the talar body.  Disruption of this artery may lead to osteonecrosis of the medial body and subsequent collapse into varus.  This is most commonly seen with talar body fractures but may be seen in Hawkins type 3 talar neck fractures.  The artery of the tarsal sinus arises from the dorsalis pedis, lateral malleolar, and perforating peroneal arteries.  The peroneal artery anastomoses with the calcaneal branches of the posterior tibial artery to form a plexus of vessels that supplies the posterior tubercle of the talus.  Disruption of this artery would not result in collapse of the medial body, and thus would not lead to a varus deformity.

 

REFERENCES: Halibruton RA, Sullivan CR, Kelly PJ, et al: The extra-osseous and intra-osseous blood supply of the talus.  J Bone Joint Surg Am 1958;40:1115.

Mulfinger GL, Trueta J: The blood supply of the talus.  J Bone Joint Surg Br 1970;52:160-167.

 

 

 

 

58.      Which of the following is considered the most useful screening method for the evaluation of protective foot sensation in a patient with diabetes mellitus?

 

1-         Sharp two-point discrimination

2-         Light touch sensation

3-         Hot and cold sensation

4-         Vibratory sensation

5-         5.07 Semmes-Weinstein monofilament sensation

 

PREFERRED RESPONSE: 5

 

DISCUSSION: Patients with diabetes mellitus should be screened for the presence of protective foot sensation.  In the absence of protective foot sensation, patients are at increased risk for the development of neuropathic ulcerations and neuropathic arthropathy.  The most reliable screening tool for the presence of protective sensation is the ability to feel the 5.07 Semmes-Weinstein monofilament.

 

REFERENCES: Pinzur MS, Shields N, Trepman E, Dawson P, Evans A: Current practice patterns in the treatment of Charcot foot.  Foot Ankle Int 2000;21:916-920.

Smith KD, Emerzian GJ, Petrov O: A comparison of calibrated and non-calibrated 5.07 nylon monofilaments.  Foot Ankle Int 2000;21:852-855.

 

 

 

 

59.      A 17-year-old high school track athlete has had progressive midfoot pain for the past 3 weeks that prevents him from running.  Examination reveals pain over the tarsal navicular.  Radiographs are normal, but a CT scan reveals a nondisplaced sagittally oriented fracture line.  Management should consist of

 

1-         an orthosis and an immediate return to running.

2-         no running for 6 weeks and use of a bone stimulator at night.

3-         a University of California Biomechanics Laboratory (UCBL) orthosis and a gradual return to running.

4-         immobilization in a short leg cast with no weight bearing for 6 to 8 weeks.

5-         open reduction and internal fixation.

 

PREFERRED RESPONSE: 4

 

DISCUSSION: The patient has a nondisplaced stress fracture of the tarsal navicular.  Weight bearing is associated with a high rate of nonunion; therefore, management should consist of immobilization and no weight bearing for 8 weeks.  Delayed union or nonunion is treated by excision of sclerotic fracture margins and bone grafting, with or without internal fixation.  Generally, CT should be repeated to document healing before permitting a return to sports.

 

REFERENCES: Beaty JH (ed): Orthopaedic Knowledge Update 6.  Rosemont, IL, American Academy of Orthopaedic Surgeons, 1999, pp 597-612.

Torg J, Pavlov H, Cooley LH, et al: Stress fractures of the tarsal navicular: A retrospective review of twenty-one cases.  J Bone Joint Surg Am 1982;64:700-712.

 

60.      A construction worker sustained a comminuted calcaneus fracture 2 years ago.  He now reports progressive hindfoot pain with the recent onset of anterior ankle pain.  A lateral hindfoot radiograph is shown in Figure 31.  Treatment should consist of

 

1-         in situ subtalar fusion.

2-         tibiotalocalcaneal arthrodesis using an intramedullary rod.

3-         ankle arthroscopy with debridement of osteophytes.

4-         subtalar distraction bone block arthrodesis.

5-         lateral wall ostectomy of the calcaneus.

 

PREFERRED RESPONSE: 4

 

DISCUSSION: The patient has subtalar arthrosis, a loss of heel height with anterior ankle impingement.  The mechanics of the ankle are impaired, and dorsiflexion is painful and limited.  The talar declination angle is measured by drawing a line through the longitudinal axis of the talus and the plane of support of the foot on a weight-bearing lateral radiograph.  Anterior impingement is suggested with any value below 20 .  By performing a distraction arthrodesis through the subtalar joint, the normal declination of the talus is reestablished, eliminating the anterior ankle impingement.  Tibiotalocalaneal fusion would be inappropriate because the patient does not have arthritic symptoms in the ankle.  Ankle arthroscopy or in situ arthrodesis would not reestablish appropriate ankle mechanics, and the osteophytes would be prone to redevelop.  Lateral wall ostectomy may help with impingement at the level of the fibula or the lateral ankle but would provide no benefit to anterior ankle impingement.

 

REFERENCES: Carr JB, Hansen ST, Benirschke SK: Subtalar distraction bone block fusion for late complications of os calcis fractures.  Foot Ankle 1988;9:81-86.

Myerson M, Quill GE Jr: Late complications of fractures of the calcaneus.  J Bone Joint Surg Am 1993;75:331.

 

 

 

 

61.      What is the most common long-term complication of the fracture shown in Figure 32?

 

1-         Nonunion

2-         Ankle arthritis

3-         Subtalar arthritis

4-         Talonavicular arthritis

5-         Posterior tibial tendinitis

 

PREFERRED RESPONSE: 3

 

DISCUSSION: The fracture pattern shown in the radiograph involves both a talar neck fracture and a talar body fracture.  The body fracture propagates into the subtalar joint, with significant risk for the development of arthritis in that surface even with an anatomic reduction.  In addition, Canale and Kelly reported a 25% incidence of malunion of talar neck fractures, with varus angulation occurring most frequently.  Of these patients, 50% required a secondary surgical procedure because of the development of degenerative joint disease of the subtalar joint.

 

REFERENCES: Canale ST, Kelly FB Jr: Fractures of the neck of the talus: Long-term evaluation of seventy-one cases.  J Bone Joint Surg Am 1978;60:143-156.

Higgins TF, Baumgaertner MR: Diagnosis and treatment of fractures of the talus: A comprehensive review of the literature. Foot Ankle Int 1999;20:595-605.

 

 

 

 

62.      A 62-year-old man has a severe pes planus and pain in the hindfoot.  Radiographs show advanced degenerative changes at the talonavicular and subtalar joints with good preservation of the ankle joint.  What is the most appropriate surgical procedure to alleviate his pain?

 

1-         Flexor digitorum longus transfer with calcaneal osteotomy

2-         Lateral column lengthening with tendon transfer

3-         Subtalar arthrodesis

4-         Triple arthrodesis

5-         Isolated talonavicular fusion

 

PREFERRED RESPONSE: 4

 

DISCUSSION: Once degenerative changes have occurred, soft-tissue procedures are not indicated.  Triple arthrodesis is the treatment of choice for adult-acquired flatfoot.  Isolated fusion of the subtalar or talonavicular joint will not be sufficient to correct the problem.

 

REFERENCES: Lutter LD, Mizel MS, Pfeffer GB (eds): Orthopaedic Knowledge Update: Foot and Ankle. Rosemont, IL, American Academy of Orthopaedic Surgeons, 1994, pp 269-282.

Graves SC, Stephenson K: The use of subtalar and triple arthrodesis in the treatment of posterior tibial tendon dysfunction.  Foot Ankle Clin 1997;2:319.

 

 

 

 

63.      A 46-year-old woman reports pain and a shortened appearance of her toe after undergoing a Keller resection arthroplasty 2 years ago for hallux rigidus.  Examination reveals mild swelling and motion limited to 25 degrees at the metatarsophalangeal joint.  Radiographs show large dorsal osteophytes on the first metatarsal head, 50% resection of the proximal phalanx, and complete loss of the metatarsophalangeal joint space.  Which of the following is considered the most reliable procedure to improve her pain and the appearance of her toe?

 

1-         Silastic arthroplasty

2-         Cheilectomy and soft-tissue interposition arthroplasty

3-         Moberg phalangeal dorsiflexion osteotomy

4-         Bone graft interposition arthrodesis

5-         Waterman first metatarsal dorsal osteotomy

 

PREFERRED RESPONSE: 4

 

DISCUSSION: Because the patient has significant arthritis, arthrodesis is the treatment of choice.  Adding a bone graft will prevent further shortening and add length to her toe, resulting in improved cosmesis.  A cheilectomy will not alleviate her arthritis pain.  The toe is too short for an effective Moberg phalangeal dorsiflexion osteotomy.  A Waterman first metatarsal dorsal osteotomy will not address the degenerative joint disease or shortening.  Silastic arthroplasty may help, but there is the risk of additional problems with foreign body reaction and a significant risk of failure known to occur with Silastic materials.

 

REFERENCES: Myerson MS, Schon LC, McGuigan FX, Oznur A:Result of arthrodesis of the hallux metatarsophalangeal joint using bone graft for restoration of length.  Foot Ankle Int 2000;21:297-306.

Mann RA, Coughlin MJ: Adult hallux valgus, in Coughlin MJ, Mann RA (eds): Surgery of the Foot and Ankle, ed 7.  St Louis, MO, Mosby, 1999, pp 252-253.

Mizel MS, Miller RA, Scioli MW (eds): Orthopaedic Knowledge Update: Foot and Ankle 2.  Rosemont, IL, American Academy of Orthopaedic Surgeons, 1998, pp 293-303.

 

 

 

 

64.      Which of the following is considered the most appropriate shoe modification following transmetatarsal amputation?

 

1-         Foam filling of the forefoot void

2-         Custom last shoe of a smaller size

3-         Solid ankle polypropylene ankle-foot orthosis

4-         Silicone partial foot prosthesis with cosmetic toes

5-         Cushioned molded insole and toe filler over a carbon fiber footplate

 

PREFERRED RESPONSE: 5

 

DISCUSSION: Most patients who undergo transmetatarsal amputation do not require custom shoe wear or an orthosis above the ankle.  A molded toe filler is used to prevent excessive shear that can lead to ulceration.  Use of a soft toe filler without stiffening of the sole results in excessive flexibility from the shortened lever arm, which reduces the efficiency of gait.  A firm footplate or carbon fiber base adds rigidity to aid in push-off.  A rocker bottom also may be added to the shoe.

 

REFERENCES: Philbin TM, Leyes M, Sferra JJ, Donley BG:  Orthotic and prosthetic devices in partial foot amputations.  Foot Ankle Clin 2001;6:215-228.

Marks RM: Mid-foot/mid-tarsus amputations.  Foot Ankle Clin 1999;4:1-16.

 

 

 

 

65.      A 35-year-old man has had a mass on the bottom of his foot for the past 6 months.  He reports that initially the mass was exquisitely painful but now is minimally tender.  Examination reveals a 2.5- x 2.0-cm firm, noncompressible, nonmobile mass contiguous with the plantar fascia in the distal arch.  The mass is particularly prominent with passive dorsiflexion of the ankle and toes.  What is the best course of action?

 

1-         Observation

2-         Incisional biopsy

3-         Excisional biopsy

4-         MRI

5-         Ultrasound

 

PREFERRED RESPONSE: 1

 

DISCUSSION: The history is most consistent with a plantar fibroma.  The nodules typically are located within the substance of the plantar aponeurosis.  The clinical appearance is usually diagnostic without the need for advanced imaging studies.  While the lesion may be prominent and painful to direct palpation, the anatomic location is usually off of the weight-bearing surface.  Observation with or without an accommodative orthotic is the treatment of choice.  Recurrence is common following attempted excision.

 

REFERENCES: Sammarco GJ, Mangone PG: Classification and treatment of plantar fibromatosis.  Foot Ankle Int 2000;21:563-569.

Durr HR, Krodel A, Trouillier H, Lienemann A, Refior HJ: Fibromatosis of the plantar fascia: Diagnosis and indications for surgical treatment.  Foot Ankle Int 1999;20:13-17.

 

 

 

 

66.      A 25-year-old woman has significant pain and swelling in her left ankle after falling off her bicycle.  Examination reveals that she is neurovascularly intact.  Radiographs are shown in Figures 33a through 33c.  What is the next most appropriate step in management?

 

1-         Closed reduction and casting

2-         Open reduction and internal fixation of the ankle fracture

3-         Open reduction and internal fixation of the ankle fracture with syndesmosis fixation

4-         Percutaneous pinning of the ankle fracture

5-         Pins in plaster immobilization

 

PREFERRED RESPONSE: 3

 

DISCUSSION: The radiographs show a displaced ankle fracture with widening of the syndesmosis.  Open reduction and internal fixation is indicated with fixation of the mortise with syndesmotic screws.

 

REFERENCES: Wuest TK: Injuries to the distal lower extremity syndesmosis.  J Am Acad Orthop Surg 1997;5:172-181.

Harper MC: Delayed reduction and stabilization of the tibiofibular syndesmosis.  Foot Ankle Int 2001;22:15-18.

 

 

 

 

67.      A 55-year-old woman with type I diabetes mellitus has a chronic ulcer over the dorsum of her right foot and reports forefoot pain.  Examination reveals 1- x 2-cm nondraining ulcer over the dorsum of the foot.  The patient has 1-2+ pain with compression of the foot and ankle.  She has a weakly palpable posterior tibial pulse and an absent dorsalis pedis pulse.  There is no erythema, cellulitis, or drainage.  Radiographs are normal.  Which of the following diagnostic studies should be obtained?

 

1-         MRI

2-         Doppler arterial study

3-         Arteriography

4-         Electromyography

5-         Triple phase bone scan (technetium Tc 99m)

 

PREFERRED RESPONSE: 2

 

DISCUSSION: The presence of a dorsal ulcer in the presence of weak or absent pulses strongly suggests the possibility of arterial insufficiency.  The best initial noninvasive study to assess for ischemia is the Doppler arterial study.  A determination of the vascular status is of a greater priority than an assessment for infection or neuropathy because of the location and presentation of the ulcer.  If ankle pressures are less than 45 mm Hg, there is a high risk that these lesions will not heal without revascularization.

 

REFERENCES: Wagner FW Jr: The dysvascular foot: A system for diagnosis and treatment.  Foot Ankle 1981;2:64-122.

Apelqvist J, Castenfors J, Larson J, Stenstrom A, Agardh CD: Prognostic value of systolic ankle and toe blood pressure levels in outcome of diabetic foot ulcer.  Diabetes Care 1989;12:373-378.

 

 

 

 

68.      A 57-year-old woman with diabetes mellitus has purulent drainage from a lateral incision after undergoing open reduction and internal fixation of a displaced ankle fracture 10 days ago.  Examination reveals moderate erythema and a foul odor coming from the wound.  Cultures are obtained.  What is the next most appropriate step in management?

 

1-         Oral cephalosporin

2-         IV cephalexin and dressing changes

3-         Betadine dressing and a short leg cast

4-         Debridement of the wound and removal of the hardware

5-         Debridement of the wound and maintenance of the hardware

 

PREFERRED RESPONSE: 5

 

DISCUSSION: Early postoperative wound infections after open reduction and internal fixation should be treated with aggressive debridement and maintenance of stability of the fracture.  If infection persists following healing of the fracture, the hardware should be removed.

 

REFERENCES: Carragee EJ, Csongradi JJ, Bleck EE: Early complications in the operative treatment of ankle fractures: Influence of delay before operation.  J Bone Joint Surg Br 1991;73:79-82.

Blotter RH, Connolly E, Wasan A, Chapman MW: Acute complications in the operative treatment of isolated ankle fractures in patients with diabetes mellitus.  Foot Ankle Int 1999;20:687-694.

 

 

 

 

69.      A 67-year-old woman has had pain in the area of the metatarsal heads and toes bilaterally for the past 18 months.  She describes a diffuse discomfort and a constant burning sensation.  She notes that the area feels swollen.  Examination reveals that her pulses are normal, and there is no frank swelling or focal tenderness.  What is the most likely diagnosis?

 

1-         Peripheral vascular disease

2-         Morton’s neuroma

3-         Stress fracture

4-         Peripheral neuropathy

5-         Freiberg’s infraction

 

PREFERRED RESPONSE: 4

 

DISCUSSION: Patients with peripheral neuropathy will often initially see an orthopaedic surgeon and report symptoms of burning, numb, dead, or wooden feet.  A simple diagnostic evaluation with a tuning fork (to test vibratory sensibility) or use of the Semmes-Weinstein monofilaments will help make the diagnosis.

 

REFERENCES: Mizel MS, Miller RA, Scioli MW (eds): Orthopaedic Knowledge Update: Foot and Ankle 2.  Rosemont, IL, American Academy of Orthopaedic Surgeons, 1998, pp 113-121.

Gorson KC, Ropper AH: Idiopathic distal small fiber neuropathy.  Acta Neurol Scand 1995;92:376-382.

 

 

 

 

70.      A 19-year-old woman has had a painful prominence on the lateral border of her fifth metatarsal head since she was a young girl.  Nonsurgical management, including the use of a wide toe box shoe, has failed to provide relief.  Examination reveals a callus over the lateral prominence and on the plantar portion as well.  A clinical photograph and a radiograph are shown in Figures 34a and 34b.  Treatment should consist of

 

1-         chevron osteotomy of the fifth metatarsal head, with capsular plication and release of the medial collateral ligament.

2-         metatarsal head excision with soft-tissue interposition.

3-         “floating” distal oblique osteotomy.

4-         oblique biplanar diaphyseal fifth metatarsal osteotomy.

5-         proximal crescentic osteotomy of the fifth metatarsal base.

 

PREFERRED RESPONSE: 4

 

DISCUSSION: The type of deformity described is a type 2 bunionette.  There is often a congenital component to this deformity.  The bowing of the fifth shaft differentiates a large intermetatarsal angle from a type 3 deformity.  A distal chevron osteotomy corrects 1 degree in the intermetatarsal angle for every 1-mm shift.  Because of limitations in the width of the fifth metatarsal neck, the allowable shift is generally 3 to 4 mm.  This shift will not compensate for the large intermetatarsal angle.  The floating osteotomy has a high rate of delayed union/nonunion and a low satisfaction rate.  Metatarsal head excision has a high complication rate, including severe shortening, transfer metatarsalgia, stiffness, and pain.  A more proximal procedure is necessary to correct the large intermetatarsal angle and the lateral bowing.  The osteotomy of choice is a diaphyseal shaft osteotomy.  Because this patient has a plantar callosity and a lateral callosity, the osteotomy is angled superiorly to elevate the fifth shaft with the shift, eliminating overload of the plantar metatarsal head and subsequent callus formation.

 

REFERENCES: Shereff MJ, Yang QM, Kummer FJ, Frey CC, Greenidge N: Vascular anatomy of the fifth metatarsal.  Foot Ankle 1991;11:350-353.

Coughlin MJ: Treatment of bunionette deformity with longitudinal diaphyseal osteotomy with distal soft tissue repair.  Foot Ankle 1991;11:195-203.

Kitaoka HB, Holiday AD Jr: Metatarsal head resection for bunionette: Long-term follow-up.  Foot Ankle 1991;11:345-349.

 

 

 

 

71.       A 61-year-old woman has increasing pain in her left great toe.  She states that she has had discomfort for years but now has pain with all shoe wear.  A radiograph is shown in Figure 35.  To provide the most predictable pain-free result, treatment should consist of

 

1-         distal chevron bunionectomy.

2-         proximal first metatarsal osteotomy with distal soft-tissue realignment.

3-         dorsal cheilectomy of the first metatarsophalangeal joint.

4-         arthrodesis of the first metatarsophalangeal joint.

5-         silicone arthroplasty.

 

PREFERRED RESPONSE: 4

 

DISCUSSION: Because the patient has a hallux valgus with increased intermetatarsal and hallux valgus angles and advanced degenerative arthritis of the joint, arthrodesis of the first metatarsophalangeal joint will provide the most predictable pain-free result.  An attempt to correct the bunion with a bunionectomy or osteotomy would most likely fail.  The hallux valgus and advanced degenerative changes put the foot beyond the indications for a cheilectomy.  Long-term results with silicone arthroplasty have been disappointing.

 

REFERENCES: Mann RA: Disorders of the first metatarsophalangeal joint.  J Am Acad Orthop Surg 1995;3:34-43.

Coughlin MJ, Abdo RV: Arthrodesis of the first metatarsophalangeal joint with Vitallium plate fixation.  Foot Ankle Int 1994;15:18-28.

 

 

 

 

72.      The most favorable outcomes from release of the tarsal tunnel are in patients who have which of the following findings?

 

1-         Associated space-occupying lesion

2-         Dorsal midfoot pain

3-         Abnormal preoperative electrodiagnostic studies

4-         Compromised soft-tissue sleeve

5-         Intrinsic weakness and atrophy

 

PREFERRED RESPONSE: 1

 

DISCUSSION: Numerous causes of tarsal tunnel syndrome have been reported.  The most favorable outcomes from release of the tarsal tunnel are in patients who have a space-occupying lesion (eg, ganglion, lipoma, or neurilemoma).  While electrodiagnostic studies may be abnormal preoperatively, there is a low correlation between clinical outcome and electromyographic findings.  Intrinsic weakness is a late finding in long-standing nerve dysfunction.

 

REFERENCES: Beskin JL: Nerve entrapment syndromes of the foot and ankle.  J Am Acad Orthop Surg 1997;5:261-269.

Mizel MS, Miller RA, Scioli MW (eds): Orthopaedic Knowledge Update: Foot and Ankle 2.  Rosemont, IL, American Academy of Orthopaedic Surgeons, 1998, pp 101-111.

 

 

 

 

73.      An active 48-year-old woman has had progressive retrocalcaneal pain for the past 2 years.  She reports that an injection into the retrocalcaneal bursa 3 weeks ago provided relief, but she now has swelling and weakness after tripping on the stairs 3 days ago.  The Thompson test is positive.  A radiograph is shown in Figure 36.  What is the next most appropriate step in management?

 

1-         MRI to rule out a complete rupture

2-         Ultrasound of the tendon apposition

3-         Open reduction and internal fixation of the calcaneal fracture

4-         Symptomatic care and physical therapy

5-         Surgical repair with tendon debridement and flexor hallucis longus transfer

 

PREFERRED RESPONSE: 5

 

DISCUSSION: The patient’s long-standing symptoms and radiograph indicate a chronic insertional Achilles tendinopathy that has progressed to complete rupture.  This situation is best treated with tendon debridement and repair, often requiring supplementation graft from the flexor hallucis longus.  MRI could provide additional information on the quality of the Achilles tendon, but neither MRI nor ultrasound is necessary to make a diagnosis or determine the surgical indication.  Conservative management will be unpredictable with a chronic degenerative tendon injury.

 

REFERENCES: Myerson MS, McGarvey W: Disorders of the Achilles tendon: Insertion and Achilles tendinitis.  Instr Course Lect 1999;48:211-218.

Wilcox DK, Bohay DR, Anderson JG: Treatment of chronic Achilles tendon disorders with flexor hallucis longus tendon transfer/augmentation.  Foot Ankle Int 2000;21:1004-1010.

Mizel MS, Miller RA, Scioli MW (eds): Orthopaedic Knowledge Update: Foot and Ankle 2.  Rosemont, IL, American Academy of Orthopaedic Surgeons, 1998, pp 253-277.

 

 

 

 

74.      A 47-year-old woman has a right bunion that has been symptomatic despite modifications in shoe wear.  She requests surgical correction.  An AP radiograph is shown in Figure 37.  Treatment should consist of

 

1-         distal chevron bunionectomy.

2-         osteotomy of the proximal first metatarsal with distal soft-tissue realignment.

3-         a Keller bunionectomy.

4-         arthrodesis of the first metatarsophalangeal joint.

5-         double osteotomy of the first metatarsal with distal soft-tissue realignment.

 

PREFERRED RESPONSE: 2

 

DISCUSSION: Because the radiograph reveals an intermetatarsal angle of greater than 15 degrees and an incongruent metatarsophalangeal joint, the treatment of choice is a proximal first metatarsal osteotomy with distal soft-tissue realignment.  A distal chevron procedure would not correct this degree of deformity.  A Keller procedure is reserved for a less active elderly individual.  Arthrodesis is appropriate for a patient with advanced arthritis of the metatarsophalangeal joint.  The double osteotomy is reserved for the congruent metatarsophalangeal joint with hallux valgus.

 

REFERENCES: Coughlin MJ, Carlson RE: Treatment of hallux valgus with an increased distal metatarsal articular angle: Evaluation of double and triple first ray osteotomies.  Foot Ankle Int 1999;20:762-770.

Coughlin MJ: Hallux valgus.  Instr Course Lect 1997;46:357-391.

 

 

 

 

75.      A 68-year-old woman stepped on a needle while walking barefoot 10 days ago.  She is not certain but thinks it is imbedded in her foot, and she notes local tenderness at the puncture site and drainage.  Her primary care physician has been treating her with oral antibiotics.  A plain radiograph is shown in Figure 38.  What is the best course of action?

 

1-         IV antibiotics

2-         Continued oral antibiotics

3-         Removal of the foreign body in the surgical suite with fluoroscopy

4-         Removal of the foreign body in the office under local anesthesia

5-         CT to localize the foreign body

 

PREFERRED RESPONSE: 3

 

DISCUSSION: Based on the radiographic findings, the patient has a metallic foreign body in her foot that is consistent with a needle.  She has local infection secondary to the continued presence of the foreign body.  CT is not necessary to localize the foreign body as it is adequately visualized on the plain radiographs.  The infection cannot be adequately treated until the foreign body is removed.  Attempted removal of foreign bodies without proper anesthesia and fluoroscopy frequently results in frustration because of the inability to localize the foreign body.  Removal in a surgical suite with proper anesthesia and fluoroscopy is the preferred option.  Once the foreign body is removed, the local infection will resolve rapidly.

 

REFERENCES: Combs AH, Kernek CB, Heck DA: Orthopedic grand rounds: Retained wooden foreign body in the foot detected by computed tomography.  Orthopedics 1986;9:1434-1435.

Markiewitz AD, Karns DJ, Brooks PJ: Late infections of the foot due to incomplete removal of foreign bodies: A report of two cases.  Foot Ankle Int 1994;15:52-55.

 

 

 

 

76.      A 50-year-old woman who underwent a joint replacement of the hallux metatarsophalangeal joint 6 months ago now has pain and swelling about the great toe.  Radiographs are shown in Figures 39a and 39b.  What is the next most appropriate step in management?

 

1-         Removal of the implant alone

2-         Amputation of the hallux

3-         Revision of the implant with a Silastic joint replacement

4-         Arthrodesis of the hallux metatarsophalangeal joint with interposition bone graft

5-         Rocker bottom steel shank shoe with orthosis

 

PREFERRED RESPONSE: 4

 

DISCUSSION: The radiographs show displacement of the prosthesis, and there has been large amounts of bone resected to insert the implant.  Arthrodesis is indicated with interposition bone graft to stabilize the joint and restore length to the first ray.

 

REFERENCE: Myerson MS: Foot and Ankle Disorders.  Philadelphia, PA, WB Saunders, 2000, pp 265-266.

 

 

 

77.       What is the most common foot and ankle deformity in patients with arthrogryposis?

 

1-         Talipes eqinovarus

2-         Metatarsus adductus

3-         Brachymetatarsia

4-         Pes planus

5-         Tarsal coalition

 

PREFERRED RESPONSE: 1

 

DISCUSSION: Clubfoot (talipes equinovarus) in patients with arthrogryposis is a rigid and resistant deformity.  However, multiple studies document limited success with nonsurgical management.  Manipulation and casting are generally a preliminary treatment before surgery; successful correction will most like require a talectomy.  

 

REFERENCES: Guidera KJ, Drennan JC: Foot and ankle deformities in arthrogryposis multiplex congenita.  Clin Orthop 1985;194:93-98.

Handelsman JE, Badalamente MA: Neuromuscular studies in clubfoot.  J Pediatr Orthop 1981;1:23-32.

Dias LS, Stern LS: Talectomy in the treatment of resistant talipes equinovarus deformity in myelomeningocele and arthrogryposis.  J Pediatr Orthop 1987;7:39-41.

 

 

 

78.      A 16-year-old girl has had pain and swelling along the medial arch of her left foot for the past 3 months.  She also reports pain from shoe wear and while running.  Nonsteroidal anti-inflammatory drugs have failed to provide relief.  Radiographs are shown in Figures 40a through 40c.  What is the next most appropriate step in management?

 

1-         Resection of the tarsal coalition

2-         Open reduction and internal fixation of the navicular fracture

3-         Surgical resection of the accessory navicular

4-         Corticosteroid injection

5-         Custom-molded orthosis

 

PREFERRED RESPONSE: 5

 

DISCUSSION: Nonsurgical management of a symptomatic accessory navicular should be attempted prior to surgery.  Good relief is often obtained with a semi-rigid orthosis with a medial arch support.

 

REFERENCES: Myerson MS: Foot and Ankle Disorders.  Philadelphia, PA, WB Saunders, 2000, p 655.

Bennett GL, Weiner DS, Leighley B: Surgical treatment of symptomatic accessory tarsal navicular.  J Pediatr Orthop 1990;10:445-449.

 

 

 

 

79.      A 28-year-old man was shot in the foot with a .22 caliber handgun approximately 2 hours ago.  Examination reveals an entrance wound dorsally and a plantar exit wound.  The foot is neurovascularly intact.  Radiographs reveal a nondisplaced fracture of the third metatarsal.  Soft-tissue management for this injury should consist of

 

1-         surface debridement and pulsatile lavage.

2-         wide debridement and internal fixation.

3-         extensive prophylactic compartment fasciotomy.

4-         immediate arteriography.

5-         observation and IV antibiotics for 7 weeks.

 

PREFERRED RESPONSE: 1

 

DISCUSSION: The patient has sustained a low-velocity, low-caliber gunshot wound to the foot.  Because the injury occurred within a period of 8 hours, this is classified as a type I wound.  Several studies support the use of surface debridement, cleansing, and sterile dressings as the treatment of choice.  More aggressive measures are reserved for high-velocity injuries and shotgun injuries.

 

REFERENCES: Brettler D, Sedlin ED, Mendes DG: Conservative treatment of low velocity gunshot wounds.  Clin Orthop 1979;140:26-31.

Hampton OD: The indications for debridement of gunshot bullet wounds of the extremities in civilian practice.  J Trauma 1961;1:368-372.

Marcus NA, Blair WF, Shuck JM, Omer GE Jr: Low-velocity gunshot wounds to extremities.  J Trauma 1980;20:1061-1064.

 

 

 

 

80.      The photomicrograph seen in Figure 41 shows which of the following conditions?

 

1-         Synovial cell sarcoma

2-         Giant cell tumor

3-         Aneurysmal bone cyst

4-         Osteosarcoma

5-         Eosinophilic granuloma

 

PREFERRED RESPONSE: 1

 

DISCUSSION: The photomicrograph shows a synovial cell sarcoma with a characteristic histology of a biphasic pattern of pleomorphic spindle cells and well-differentiated cuboidal to columnar cells forming gland-like spaces.  The glandular zones contain mucous-like material that stains positively with periodic acid Schiff.  Microscopic calcifications are usually found.  Synovial cell sarcoma has a high rate of local recurrence as well as metastases.  It is the most common malignancy found in the foot.

 

REFERENCES: Krall RA, Kostianovsky M, Patchefsky AS: Synovial sarcoma: A clinical, pathological and ultrastructural study of 26 cases supporting the recognition of a monophasic variant.  Am J Surg Pathol 1981;5:137-151.

Wright PH, Sim FH, Soule EH, Taylor WF: Synovial sarcoma.  J Bone Joint Surg Am 1982;64:112-122.

 

 

 

 

81.       A 33-year-old man had his foot run over by a forklift 1 hour ago.  Examination reveals that the head of the fifth metatarsal is extruded through the plantar aspect of the foot.  The foot is severely swollen and pale, there is no sensation in the toes, and the pulses are not palpable.  Radiographs are shown in Figures 42a and 42b.  Emergent management should consist of

 

1-         immediate transmetatarsal amputation.

2-         immediate below-knee amputation.

3-         application of a splint and observation.

4-         fasciotomy and fracture fixation.

5-         fasciotomy without fracture fixation.

 

PREFERRED RESPONSE: 4

 

DISCUSSION: Following a severe crush injury, the patient has an acute compartment syndrome.  Even though there is an open fracture, this is not sufficient to decompress the compartment syndrome.  Therefore, splinting and observation are not appropriate.  The surgical treatment of choice is fasciotomy with fixation of the multiple fractures.  A primary amputation is not indicated because there is potential for salvage of this devastating injury.

 

REFERENCES: Fakhouri AJ, Manoli A II: Acute foot compartment syndromes.  J Orthop Trauma 1992;6:223-228.

Myerson MS: Management of compartment syndromes of the foot.  Clin Orthop 1991;271:239-248.

Ziv I, Mosheiff R, Zeligowski A, Liebergal M, Lowe J, Segal D: Crush injuries of the foot with compartment syndrome: Immediate one-stage management.  Foot Ankle 1989;9:185-189.

 

 

 

 

82.      A 2-year-old boy has been referred for musculoskeletal evaluation.  Examination reveals shortened proximal limbs, hip and knee flexion contractures, an abducted thumb, and ear abnormalities.  His parents are concerned about his deformed feet.  What is the most common foot deformity associated with this patient’s diagnosis?

 

1-         Pes calcaneovalgus

2-         Fixed pes planovalgus

3-         Rigid equinovarus

4-         Metatarsus adductus

5-         Skewfoot (forefoot adduction and heel valgus)

 

PREFERRED RESPONSE: 3

 

DISCUSSION: The patient has diastrophic dysplasia.  Affected individuals have rhizomelic short stature, cauliflower ears, severe joint contractures (especially knees and hips), hitchhiker’s thumb, and a cleft palate.  The most common foot abnormality is a rigid equinovarus deformity.  Surgical results are poorer than those for idiopathic clubfeet and often require bony procedures or talectomy.

 

REFERENCES: Ryoppy S, Poussa M, Merikanto J, Marttinen E, Kaitila I: Foot deformities in diastrophic dysplasia: An analysis of 102 patients.  J Bone Joint Surg Br 1992;74:441-444.

Bussett GS: The osteochondrodysplasias, in Morrissy RT, Weinstein S (eds): Lovell and Winter’s Pediatric Orthopaedics, ed 4.  Philadelphia, PA, Lippincott-Raven, 1996, p 219.

 

 

 

 

83.      A 31-year-old woman has a history of a painful ankle that has failed to respond to conservative management.  She has associated night pain that is relieved with nonsteroidal anti-inflammatory drugs.  MRI and technetium Tc 99m scans are consistent with an osteoid osteoma.  Management should now consist of

 

1-         curettage or en bloc resection.

2-         allograft limb salvage.

3-         injection with 5% phenol.

4-         radiation therapy.

5-         hormonal therapy.

 

PREFERRED RESPONSE: 1

 

DISCUSSION: Surgical curettage or en bloc resection is the treatment of choice for osteoid osteoma.  Night pain and relief of symptoms with nonsteroidal anti-inflammatory drugs are classic findings for osteoid osteoma.

 

REFERENCES: Donley BG, Philbin T, Rosenberg GA, Schils JP, Recht M: Percutaneous CT guided resection of osteoid osteoma of the tibial plafond.  Foot Ankle Int 2000;21:596-598.

Kenzora JE, Abrams RC: Problems encountered in the diagnosis and treatment of osteoid osteoma of the talus.  Foot Ankle 1981;2:172-178.

Shereff MJ, Cullivan WT, Johnson KA: Osteoid-osteoma of the foot.  J Bone Joint Surg Am 1983;65:638-641.

 

 

 

 

84.      A 13-year-old girl has had pain in her ankle and difficulty with sporting activities for the past 6 months.  Nonsteroidal anti-inflammatory drugs and use of a short leg cast have provided minimal relief.  A radiograph and MRI scan are shown in Figures 43a and 43b.  What is the next most appropriate step in treatment?

 

1-         Ankle ligament repair

2-         Resection of the accessory navicular

3-         Resection of the talocalcaneal coalition

4-         Subtalar arthrodesis

5-         Triple arthrodesis

 

PREFERRED RESPONSE: 3

 

DISCUSSION: The MRI scan shows an obvious talocalcaneal coalition of the medial facet.  Because nonsurgical management has failed, surgical resection of the coalition is indicated.  Arthrodesis would be indicated only if resection fails to relieve pain or if advanced degeneration of the hindfoot joints is present.

 

REFERENCES: McCormack TJ, Olney B, Asher M: Talocalcaneal coalition resection: A 10-year follow-up.  J Pediatr Orthop 1997;17:13-15.

Thometz J: Tarsal coalition.  Foot Ankle Clin 2000;5:103-118.

 

85.      A 16-year-old female dancer has persistent posterior ankle pain, particularly after a vigorous dancing schedule.  Examination reveals tenderness both posteromedially and posterolaterally.  MRI scans are seen in Figures 44a and 44b.  What is the most likely diagnosis?

 

1-         Posterior tibial tendinitis

2-         Tarsal tunnel syndrome

3-         Os trigonum impingement syndrome

4-         Insertional tendinitis of the Achilles tendon

5-         Osteochondritis dissecans of the talus

 

PREFERRED RESPONSE: 3

 

DISCUSSION: Posterior ankle impingement or os trigonum syndrome is well described in dancers, and it is often associated with flexor hallucis longus tendinitis.  High-quality MRI imaging will reveal the inflammation about the os trigonum and flexor hallucis longus tendinitis.

 

REFERENCES: Mizel MS, Miller RA, Scioli MW (eds): Orthopaedic Knowledge Update: Foot and Ankle 2.  Rosemont IL, American Academy of Orthopaedic Surgeons, 1998, pp 315-332.

Hamilton WG, Hamilton  LH: Foot and ankle injuries in dancers, in Coughlin MJ, Mann RA (eds): Surgery of the Foot and Ankle, ed 7.  St Louis, MO, Mosby, 1999, pp 1225-1256.

 

 

 

 

86.      Which of the following nerves is most commonly injured during ankle arthroscopy?

 

1-         Sural

2-         Deep peroneal

3-         Saphenous

4-         Superficial peroneal

5-         Posterior tibial

 

PREFERRED RESPONSE: 4

 

DISCUSSION: The superficial peroneal nerve, which is adjacent to the location of the lateral arthroscopic portal is most commonly injured.

 

REFERENCES: Ferkel RD, Heath DD, Guhl JF: Neurological complications of ankle arthroscopy.  Arthroscopy 1996;12:200-208.

Barber CL, Click J, Britt BT: Complications of ankle arthroscopy.  Foot Ankle 1990;10:263-266.

 

 

 

87.      An obese 56-year-old woman with hypertension has had posterior heel pain for the past 6 months.  She also notes some enlargement over the posterior aspect of the heel.  Examination reproduces pain with palpation at the insertion of the Achilles tendon.  A lateral radiograph is shown in Figure 45.  What is the most likely diagnosis?

 

1-         Stress fracture of the calcaneus

2-         Painful os trigonum

3-         Insertional Achilles tendinopathy

4-         Entrapment of the calcaneal branch of the posterior tibial nerve

5-         Rupture of the Achilles tendon

 

PREFERRED RESPONSE: 3

 

DISCUSSION: The lateral radiograph shows a traction spur consistent with tendinopathy of the Achilles tendon.  There is no displacement of the spur to suggest a rupture of the Achilles tendon, and os trigonum is not seen on the radiograph.  The examination findings are not consistent with nerve entrapment.

 

REFERENCES: Schepsis AA, Wagner C, Leach RE: Surgical management of Achilles tendon overuse injuries: A long-term follow-up study.  Am J Sports Med 1994;22:611-619.

Saltzman CL, Tearse DS: Achilles tendon injuries.  J Am Acad Orthop Surg 1998;6:316-325.

 

 

 

88.      A 42-year-old woman has a history of nontraumatic ankle swelling with tenderness over the Achilles tendon and plantar fascia.  She reports that while vacationing in Connecticut 2 months ago she noted the presence of a “red bull’s eye” rash.  Management should consist of

 

1-         physical therapy.

2-         doxycycline.

3-         ciprofloxacin.

4-         nonsteroidal anti-inflammatory drugs.

5-         gentamicin.

 

PREFERRED RESPONSE: 2

 

DISCUSSION: The most likely diagnosis is Lyme disease because of the patient’s recent vacation in an area with a high risk of exposure.  The most effective treatment is doxycycline.

 

REFERENCES: Neu HC: A perspective on therapy of Lyme infection.  Ann NY Acad Sci 1988;539:314-316.

Faller J, Thompson F, Hamilton W: Foot and ankle disorders resulting from Lyme disease.  Foot Ankle 1991;11:236-238.

 

 

89.      A 50-year-old woman has a painful hallux valgus and a painful callus beneath the second metatarsal head.  A radiograph is shown in Figure 46.  To correct these problems, treatment of the great toe deformity should consist of

 

1-         excision of the base of the phalanx of the great toe with dorsiflexion osteotomy of the second metatarsal.

2-         osteotomy of the proximal phalanx of the great toe.

3-         distal first metatarsal osteotomy with soft-tissue realignment of the great toe.

4-         fusion of the metatarsal cuneiform joint with soft-tissue realignment of the first metatarsophalangeal joint.

5-         joint replacement arthroplasty of the first metatarsophalangeal joint.

 

PREFERRED RESPONSE: 4

 

DISCUSSION: The patient has a significant hallux valgus and instability of the first ray, causing transfer metatarsalgia to the second metatarsal head.  Therefore, the best procedure is fusion of the metatarsal cuneiform joint with soft-tissue realignment of the first metatarsophalangeal joint.  This procedure provides the best chance of relieving symptoms under the second metatarsal head, as well as correcting the hallux valgus.

 

REFERENCE: Sangeorzan BJ, Hansen ST Jr: Modified Lapidus procedure for hallux valgus.  Foot Ankle 1989;9:262-266.

 

 

 

 

90.      The lower extremity motor dysfunction in Charcot-Marie-Tooth disease most commonly involves which of the following muscles?

 

1-         Gastrocnemius-soleus complex

2-         Tibialis anterior

3-         Peroneus longus

4-         Flexor digitorum longus

5-         Flexor hallucis longus

 

PREFERRED RESPONSE: 2

 

DISCUSSION: The motor dysfunction in Charcot-Marie-Tooth disease involves the tibialis anterior muscle.  Charcot-Marie-Tooth disorders most commonly cause distal motor dysfunction in the foot intrinsics, anterior compartment musculature, and peroneals.  There is evidence that the peroneus brevis is affected selectively and the peroneus longus is spared.  This is based on clinical muscle testing, muscle cross-sections on MRI, and electrodiagnostic testing.

 

REFERENCES: Mann RA, Missirian J: Pathophysiology of Charcot-Marie-Tooth disease.  Clin Orthop 1988;234:221-228.

Tynan MC, Klenerman L, Helliwell TR, Edwards RH, Hayward M: Investigation of muscle imbalance in the leg in symptomatic forefoot pes cavus: A multidisciplinary study.  Foot Ankle 1992;13:489-501.

 

 

91.      Fixed hyperextension of the metatarsophalangeal joint is associated with

 

1-         dorsal subluxation of the interossei.

2-         dorsal subluxation of the lumbricals.

3-         fibrosis of the plantar plate.

4-         attenuation of the extensor longus tendon.

5-         extrinsic flexor paralysis.

 

PREFERRED RESPONSE: 1

 

DISCUSSION: Claw toe and hammer toe deformities are associated with dorsal subluxation of the interossei, which can no longer serve to flex the metatarsophalangeal joint.  The extensor digitorum longus then loses its tenodesing effect on the proximal interphalangeal and distal interphalangeal joints and works unopposed to extend the metatarsophalangeal joint and the proximal phalanx.  Without the antagonistic action of the extensor digitorum longus, the extrinsic flexors become unopposed flexors of the proximal and distal interphalangeal joints.

 

REFERENCES: Marks RM: Anatomy and pathophysiology of lesser toe deformities.  Foot Ankle Clin 1998;3:199-213.

Myerson MS, Shereff MJ: The pathological anatomy of claw and hammer toes.  J Bone Joint Surg Am 1989;71:45-49.

 

 

 

 

92.      The orthosis shown in Figure 47 is commonly used for

 

1-         hallux rigidus.

2-         midfoot arthritis.

3-         second metatarsal stress fracture.

4-         bunionette deformity.

5-         heel pain.

 

PREFERRED RESPONSE: 1

 

DISCUSSION: The orthosis shown is a carbon reinforced Morton’s extension, and it is commonly used for hallux rigidus.  It decreases motion of the first metatarsophalangeal joint and subsequently decreases pain.

 

REFERENCE: Clanton TO: Athletic injuries to the soft tissues of the foot and ankle, in Coughlin MJ, Mann RA (eds): Surgery of the Foot and Ankle, ed 7.  St Louis, MO, Mosby, 1999, vol 2, p 1185.

 

 

 

 

93.      A 14-year-old boy has medial ankle pain, progressive unilateral flatfoot deformity, and pain with most activities of daily living.  He denies any recent injury.  His parents recall that at age 7 years he sustained an injury that was treated as a sprain.  Examination reveals valgus deformity with painless, unrestricted passive motion of the ankle.  He has grossly equal limb lengths.  A radiograph of the affected ankle is shown in Figure 48a, and the contralateral ankle is shown in Figure 48b.  Management should consist of

 

1-         a University of California Biomechanics Laboratory (UCBL) orthosis.

2-         subtalar arthrodesis.

3-         physeal bar resection.

4-         tibial epiphysiodesis.

5-         closing wedge distal tibial osteotomy.

 

PREFERRED RESPONSE: 5

 

DISCUSSION: Angular deformities of the ankle can occur following physeal injury.  While an orthosis may be beneficial, the deformity is at the level of the ankle rather than the hindfoot.  An epiphysiodesis or physeal bar resection would not be indicated as the growth plates are closed.  Correction of the angular deformity should level the ankle joint and normalize the weight-bearing stresses on the ankle.  This is most easily achieved with a closing wedge distal tibial osteotomy with or without concomitant osteotomy of the fibula.

 

REFERENCES: Thompson DM, Calhoun JH: Advanced techniques in foot and ankle reconstruction.  Foot Ankle Clin 2000;5:417-442.

Ting AJ, Tarr RR, Sarmiento A, Wagner K, Resnick C: The role of subtalar motion and ankle contact pressure changes from angular deformities of the tibia.  Foot Ankle 1987;7:290-299.

Tarr RR, Resnick CT, Wagner KS, Sarmiento A: Changes in tibiotalar joint contact areas following experimentally induced tibial angular deformities.  Clin Orthop 1985;199:72-80.

 

 

 

 

94.      What is the most common organism found following a nail puncture wound through tennis shoes in a host without immunocompromise?

 

1-         Methicillin-resistant Staphylococcus aureus

2-         Escherichia coli

3-         Enterobacter

4-         Pseudomonas aeruginosa

5-         Klebsiella pneumoniae

 

PREFERRED RESPONSE: 4

 

DISCUSSION: The association of a nail puncture wound with a gram-negative infection (Pseudomonas aeruginosa) has been attributed to the local environmental factors in shoes.  Osteomyelitis is rare, occurring only in about 1% of patients.  Tetanus prophylaxis should be given if it is not up to date.  While the remaining organisms listed are periodically involved, they are more common in patients who are immunocompromised or who have diabetes mellitus.  Therefore, obtaining a culture of the infected wound is appropriate in such individuals because of the multifactorial nature of the infection.

 

REFERENCES: Green NE, Bruno J III: Pseudomonas infections of the foot after puncture wounds.  South Med J 1980;73:146-149.

Riegler HF, Routson GW: Complications of deep puncture wounds of the foot.  J Trauma 1979;19:18-22.

 

 

 

 

95.      Examination of a 28-year-old woman reveals a moderate hallux valgus deformity and a prominence of the medial eminence.  She reports that she can participate in all activities, wear 3-inch heels with minimal discomfort, and walk in a 1-inch heel with no pain.  However, she is concerned that the deformity will get worse and requests recommendations regarding surgical correction.  What is the best course of action?

 

1-         Hallux valgus correction

2-         Custom orthosis to prevent further deformity

3-         Observation only

4-         Steroid injection to decrease inflammation

5-         Extra-depth shoes

 

PREFERRED RESPONSE: 3

 

DISCUSSION: Because the patient is essentially asymptomatic, the most appropriate course of action is observation.  Prophylactic hallux valgus surgery is not medically indicated.  Steroid injection would only risk infection, as well as joint and capsule damage.  There are no data to support the use of a custom orthosis to delay the progression of a hallux valgus deformity.  Special shoe wear or an extra-depth shoe is not necessary and is unlikely to be accepted by the patient.

 

REFERENCES: Donley BG, Tisdel CL, Sferra JJ, Hall JO: Diagnosing and treating hallux valgus: A conservative approach for a common problem.  Cleve Clin J Med 1997;64:469-474.

Teitz CC, Hu SS, Arendt EA: The female athlete: Evaluation and treatment of sports-related problems.  J Am Acad Orthop Surg 1997;5:87-96.

 

 

 

 

96.      A 25-year-old man has ankle instability and a lateral foot callosity.  Radiographs are shown in Figures 49a through 49c.  Management options are best determined by the

 

1-         patient’s response to physical therapy.

2-         patient’s response to casting.

3-         patient’s response to selective injections.

4-         results of Coleman block testing.

5-         results of Semmes-Weinstein monofilament testing.

 

PREFERRED RESPONSE: 4

 

DISCUSSION: The patient has a cavovarus deformity that has resulted in lateral foot overload and stressing of the lateral ligaments.  Further treatment depends on the ability to correct the deformity.  The Coleman block test indicates whether a deformity is fixed or supple.  A supple deformity will respond to orthotic management or soft-tissue procedures, while a fixed deformity requires corrective osteotomy or fusion.  Physical therapy, casting, and injection will not address the underlying pathophysiology.  There is no indication that this is a neuropathic problem.

 

REFERENCE: Coleman SS, Chestnut WJ: A simple test for hindfoot flexibility in the cavovarus foot.  Clin Orthop 1977;123:60-62.

 

 

 

 

97.      A 17-year-old boy underwent open reduction and internal fixation of a navicular fracture 5 days ago.  A follow-up examination now reveals a tensely swollen foot with erythema and multiple skin bullae.  The patient is febrile and has marked pain with palpation of the entire forefoot and hindfoot.  What is the next step in management?

 

1-         Application of a compressive dressing and boot brace and monitoring of compartment pressures

2-         Administration of IV antibiotics and strict elevation

3-         Oral antibiotics, elevation, and close follow-up

4-         Debridement of the bullae and application of a new cast

5-         Urgent surgical debridement and IV antibiotics

 

PREFERRED RESPONSE: 5

 

DISCUSSION: Necrotizing fasciitis is a rapidly progressive soft-tissue infection with the potential to threaten both life and limb.  Patients who are immunocompromised (HIV infection, diabetes mellitus, alcohol abuse) are at increased risk.  However, any patient in the immediate postoperative phase is susceptible to wound infection.  Early detection is the key.  Necrotizing fasciitis is primarily a surgical problem that requires urgent debridement and broad-spectrum IV antibiotics.  Rapid diagnosis and prompt treatment help to reduce mortality, which may approach 30%.  Debridement of the bullae and observation are not indicated.  Although elevation and close follow-up may be warranted early on, in this patient, surgical debridement is the next step.

 

REFERENCES: Ault MJ, Geiderman J, Sokolov R: Rapid identification of group A streptococcus as the cause of necrotizing fasciitis.  Ann Emerg Med 1996;28:227-230.

McHenry CR, Piotrowski JJ, Pentrinic D, Malangoni MA: Determinants of mortality for necrotizing soft-tissue infections.  Ann Surg 1995;221:558-563.

 

 

 

 

98.      A 14-year-old girl has had mild pain and nail deformity of the great toe for the past 4 months.  A radiograph is shown in Figure 50.  What is the most likely etiology of the lesion?

 

1-         Fungal infection

2-         Ingrowth of the medial nail

3-         Benign exostosis

4-         Malignant neoplasm

5-         Reactive pyogenic granuloma

 

PREFERRED RESPONSE: 3

 

DISCUSSION: The lesion is typical of a subungual exostosis, which is most often found on the medial aspect of the great toe in children and young adults.  The diagnosis is confirmed on radiographs and usually requires excision for relief.

 

REFERENCES: Lokiec F, Ezra E, Krasin E, Keret D, Wientraub S: A simple and efficient surgical technique for subungual exostosis.  J Pediatr Orthop 2001;21:76-79.

Letts M, Davidson D, Nizalik E: Subungual exostosis: Diagnosis and treatment in children.  J Trauma 1998;44:346-349.

Davis DA, Cohen PR: Subungual exostosis: Case report and review of the literature. Pediatr Dermatol 1996;13:212-218.

 

 

 

 

99.      The third plantar intrinsic muscle layer of the foot consists of which of the following structures?

 

1-         Flexor digiti minimi, flexor hallucis brevis, adductor hallucis brevis

2-         Quadratus plantae, flexor hallucis brevis, flexor digiti minimi

3-         Quadratus plantae and plantar interosseous muscles

4-         Quadratus plantae, flexor digitorum brevis, flexor hallucis brevis

5-         Abductor hallucis, flexor digitorum brevis, flexor hallucis brevis

 

PREFERRED RESPONSE: 1

 

DISCUSSION: The plantar intrinsic muscles are divided into four layers with respect to depth from the plantar fascia.  They are (from superficial to deep): 1) abductor hallucis, flexor digitorum brevis, abductor digiti minimi; 2) quadratus plantae, lumbricals; 3) flexor digiti minimi, flexor hallucis brevis, adductor hallucis brevis; and 4) dorsal and plantar interosseous muscles.  The flexor hallucis brevis and adductor hallucis brevis originate from the midtarsal bones, encompass the sesamoids, and insert into the base of the proximal phalanx.  The adductor hallucis brevis consists of two muscle bellies forming a conjoined tendon and inserting into the lateral portion of the proximal phalanx and the lateral sesamoid.  The adductor hallucis brevis is stronger than the abductor hallucis brevis, which may contribute to hallux valgus.  The flexor digitorum minimi travels under the fifth metatarsal, arising at the base and inserting into the lateral base of the fifth proximal phalanx.

 

REFERENCE: Resch S: Functional anatomy and topography of the foot and ankle, in Myerson MS (ed): Foot and Ankle Disorders.  Philadelphia, PA, WB Saunders, 2000, pp 32-33.

 

 

 

 

100.   Which of the following results cannot be achieved with an in-shoe orthosis?

 

1-         Correction of rigid deformity

2-         Increased cushion

3-         Diminished shear forces

4-         Metatarsal head relief

5-         Redistribution of plantar forces

 

PREFERRED RESPONSE: 1

 

DISCUSSION: Depending on the type of materials used, an orthotic can be fabricated to achieve a variety of results.  While a rigid fixed deformity can be stabilized or cushioned, an orthotic will not correct a deformity that is not passively correctable.

 

REFERENCES: Mizel MS, Miller RA, Scioli MW (eds): Orthopaedic Knowledge Update: Foot and Ankle 2. Rosemont, IL, American Academy of Orthopaedic Surgeons, 1998, pp 55-64.

Bono CM, Berberian WS: Orthotic devices: Degenerative disorders of the foot and ankle.  Foot Ankle Clin 2001;6:329-340.

Buonomo LJ, Klein JS, Keiper TL: Orthotic devices: Custom-made, prefabricated, and material selection.  Foot Ankle Clin 2001;6:249-252.

 

 

 

 

FOR ALL MCQS CLICK THE LINK ORTHO MCQ BANK